Anda di halaman 1dari 232

MCQ of Davidson's principles

and practice of Medicine


22th Edition

Download and prepared by Osama M. Alsurmi

Medical spring 29th batch


Prepared by: Osama M. Alsurmi

contents
IMMUNOLOGICAL FACTORS IN DISEASE ...........................................................................................................................3

PRINCIPLES OF INFECTIOUS DISEASE ........................................................................................................................... 16

INFECTIOUS DISEASE ........................................................................................................................................................ 28

HIV INFECTIONS & AIDS .................................................................................................................................................... 62

SEXUALLY TRANSMITTED INFECTIONS ......................................................................................................................... 73

ENDOCRINE .......................................................................................................................................................................... 81

DIABETES MELLITUS ....................................................................................................................................................... 108

ALIMENTARY TRACT & PANCREATIC DISEASE ........................................................................................................... 127

LIVER& BILIARY TRACT DISEASE .................................................................................................................................. 170

RHUMATOLOGY & BONE DISEASE .................................................................................................................................. 200

777447225 1
Prepared by: Osama M. Alsurmi

777447225 2
Prepared by: Osama M. Alsurmi

Immunological factors in disease

777447225 3
Prepared by: Osama M. Alsurmi

Question 1
Which of the following is a component of the adaptive immune response in
humans?

A. Low pH of skin
B. Secretion of lysozyme
C. Colonisation with commensal bacteria
D. Phagocytes
E. T lymphocytes

The immune system is broadly divided into innate defences and adaptive responses. Innate defences are
non-specific, fast-response systems that do not exhibit memory; the acquired (or adaptive) immune
response is targeted against the specific antigen, and changes in specificity and magnitude after initial
exposure (memory). T lymphocytes are an essential component of the acquired immune response.

777447225 4
Prepared by: Osama M. Alsurmi

Question 2
A gardener cuts his finger while using a rusty knife. Over the following 24
hours, the finger becomes red, warm and painful. Which cell type is most
important in immune defence during these initial stages of infection?

A. Eosinophils
B. Neutrophils
C. T cells
D. B cells
E. Basophils

Neutrophils play an important role in the initial stages of infection, killing microorganisms directly and
non-specifically amplifying the immune response. T and B cells form part of the adaptive immune
response, which is more specific, but slower to respond.

777447225 5
Prepared by: Osama M. Alsurmi

Question 2
A gardener cuts his finger while using a rusty knife. Over the following 24
hours, the finger becomes red, warm and painful. Which cell type is most
important in immune defence during these initial stages of infection?

A. Eosinophils
B. Neutrophils
C. T cells
D. B cells
E. Basophils

Neutrophils play an important role in the initial stages of infection, killing microorganisms directly and
non-specifically amplifying the immune response. T and B cells form part of the adaptive immune
response, which is more specific, but slower to respond.

777447225 6
Prepared by: Osama M. Alsurmi

Question 4
According to the Gell and Coombs classification of hypersensitivity, which of
the following is an example of a type II (antibody-mediated) reaction?

A. Acute anaphylaxis
B. Acute cell-mediated transplant rejection
C. ABO blood transfusion reaction
D. Farmer’s lung
E. Nickel hypersensitivity

Type II reactions are characterised by antibodies to cell surface antigens. In ABO transfusion reactions,
anti-blood group antibodies bind to blood group determinants on the surface of red cells and cause cell
killing.

777447225 7
Prepared by: Osama M. Alsurmi

Question 5
A 35-year-old man presents with his third episode of meningococcal
meningitis. What is the most likely primary immune deficiency that could
account for this?

A. Complement deficiency
B. T-cell deficiency
C. Neutrophil deficiency
D. Antibody deficiency
E. Primary cytokine deficiency

Formation of the membrane attack complex of complement is essential in defence against Neisseria
meningitidis and other encapsulated bacteria.

777447225 8
Prepared by: Osama M. Alsurmi

Question 6
An 8-year-old boy presents with recurrent bacterial chest infections and
bronchiectasis. What is the most likely primary immune deficiency that
could account for this?

A. Complement deficiency
B. T-cell deficiency
C. Neutrophil function disorder
D. Antibody deficiency
E. Cytokine defect

Recurrent bacterial infection of the upper and lower respiratory tract with end-organ damage is a common
presentation of patients with primary antibody deficiency. Common organisms include Haemophilus
influenzae andStreptococcus pneumoniae.

777447225 9
Prepared by: Osama M. Alsurmi

Question 7
A 30-year-old woman, who is known to be hepatitis C-positive, develops
joint pain, microscopic haematuria and a purpuric rash. Which diagnosis
would best explain her symptoms?

A. Antiphospholipid syndrome
B. Autoimmune thrombocytopenia
C. Liver cirrhosis
D. Cryoglobulinaemia
E. Autoimmune hepatitis

The combination of joint pain, microscopic haematuria and a purpuric rash is suggestive of small-vessel
vasculitis. In patients with hepatitis C, this is most commonly caused by cryoglobulin deposition.
Autoimmune thrombocytopenia can cause a purpuric rash but does not commonly cause microscopic
haematuria or joint pain.

777447225 10
Prepared by: Osama M. Alsurmi

Question 8
Which of the following measures is least likely to form part of the
management of a patient with an antibody deficiency syndrome?

A. Long-term prophylactic antibiotics


B. Vaccination with live attenuated vaccines
C. Intravenous immunoglobulin infusion
D. Aggressive treatment of even mild infections
E. Monitoring of immunoglobulin levels

Live vaccines should always be avoided in immunocompromised patients (and patients in whom immune
deficiency is suspected) because of the risk of iatrogenic infection.

777447225 11
Prepared by: Osama M. Alsurmi

Question 9
A 48-year-old man presents with progressive visual loss and is found to
have fulminant cytomegalovirus (CMV) infection of the retina. He also has
recurrent oral candidiasis. A deficiency of which of the following immune
components is most likely to cause these problems?

A. Phagocytes
B. Complement
C. T lymphocytes
D. Antibodies
E. Neutrophils

This is a typical presentation of T-lymphocyte deficiency, which can cause increased susceptibility to
viral, fungal and mycobacterial infections. The most common cause of T-lymphocyte deficiency
worldwide is HIV infection.

777447225 12
Prepared by: Osama M. Alsurmi

Question 10
In DiGeorge syndrome, abnormal development of which of the following
structures leads to immune deficiency?

A. Thymus
B. Pituitary
C. Thyroid
D. Parathyroid
E. Spleen

DiGeorge syndrome is a congenital disorder characterised by thymic aplasia. Normally, T lymphocytes


mature in the thymus; in the absence of a thymus, T cells fail to mature and patients are susceptible to
fungal, viral and mycobacterial infections. Patients with DiGeorge syndrome also have failure of
parathyroid development, but this causes abnormal calcium metabolism and does not lead to immune
deficiency.

777447225 13
Prepared by: Osama M. Alsurmi

Question 11
Which one of the following diseases is caused by autoantibodies directed
against organ-specific antigens?

A. Hashimoto’s thyroiditis
B. Dupuytren’s contracture
C. Mixed connective tissue disease
D. Systemic lupus erythematosus
E. Light chain amyloidosis

Hashimoto’s thyroiditis is associated with autoantibodies directed at antigens within the thyroid gland,
including thyroid peroxidase, and a lymphocytic infiltrate within the gland. Mixed connective tissue
disease and systemic lupus erythematosus are associated with autoantibodies directed against widespread
antigens that are not organ-specific, including nuclear antigens. Light chain amyloidosis is associated
with high immunoglobulin levels but not with autoantibodies. Dupuytren’s contracture is a fibrotic
condition of unknown aetiology.

777447225 14
Prepared by: Osama M. Alsurmi

Question 12
Which of the following best describes the characteristic features of the
amyloid diseases?

A. Extracellular deposition of insoluble proteins


B. Intracellular deposition of insoluble proteins
C. Extracellular deposition of soluble proteins
D. Intracellular deposition of soluble proteins
E. Intracellular deposition of insoluble carbohydrates

The amyloidoses are characterised by the extracellular deposition of insoluble protein complexes.

777447225 15
Prepared by: Osama M. Alsurmi

Principles of infectious disease

777447225 16
Prepared by: Osama M. Alsurmi

Question 1
Which one of the following statements regarding infectious agents
is incorrect?

A. Bacteria possess a cell wall


B. Viruses contain either DNA or RNA
C. Prions are highly complex structures consisting of over 100 different proteins
D. Fungal cells contain a defined nucleus
E. Viruses cannot reproduce autonomously

Prions are the simplest of all infectious agents, consisting of a single protein molecule, which can catalyse
a change in a host protein. Viruses only contain one type of genetic material, either DNA or RNA, and
require a host cell (either prokaryotic or eukaryotic) in order to reproduce. Fungal cells have a defined
nucleus, in contrast to bacteria, whose nuclear material is not organised into a specific organelle.

777447225 17
Prepared by: Osama M. Alsurmi

Question 2
Which one of the following organisms is incorrectly matched with its mode
of transmission?

A. A
B. B
C. C
D. D
E. E

In contrast to hepatitis B and C, which are transmitted by sexual contact, blood transfusions and
needlestick injuries, hepatitis A is spread by the faecal–oral route.

777447225 18
Prepared by: Osama M. Alsurmi

Question 3
Which one of the following statements regarding ß-lactam antibiotics
is incorrect?

A. Benzylpenicillin, flucloxacillin and meropenem are all subtypes of this class


B. They are bactericidal antibiotics
C. Most patients with established allergy to benzylpenicillin are also allergic to
cephalosporins
D. Synergism occurs in combination with the aminoglycosides (e.g. gentamicin)
E. Potential side-effects include gastrointestinal upset and interstitial nephritis

Fewer than10% of penicillin-sensitive patients are allergic to cephalosporins.Anaphylaxis to second- and


third-generation cephalosporins is exceedingly rare, even in the presence of established penicillin allergy.
Beta-lactam antibiotics are bactericidal, meaning that they kill bacteria. They act by inhibiting synthesis
of the bacterial cell wall, which is essential for survival of bacteria in osmotically challenging
environments.

777447225 19
Prepared by: Osama M. Alsurmi

Question 4
Which one of the following antimicrobials is incorrectly matched with its
adverse side-effect?

A. A
B. B
C. C
D. D
E. E

Ketoconazole is an oral antifungal agent that can cause hepatotoxicity. ‘Grey baby syndrome’ describes
the dose-dependent cyanosis and circulatory collapse that occur in infants due to inability to conjugate
chloramphenicol and excrete it in urine. All tetracycline antibiotics bind to metallic ions in bones and
teeth, causing discoloration, and therefore should be avoided in children and in pregnancy. Tetracyclines
also photosensitise, and patients should be warned about this and advised to use sun-screen.
Erythromycin causes a cholestatic jaundice in some patients, which abates when the drug is stopped.

777447225 20
Prepared by: Osama M. Alsurmi

Question 5
Which one of the following vaccines is inactivated (as opposed to live)?

A. Pertussis
B. Measles
C. Mumps
D. BCG (tuberculosis)
E. Rubella

All of the above, apart from pertussis, are live attenuated vaccines and should not cause infection in the
immunocompetent recipient. However, they should be given with caution or avoided in those who are
immunosuppressed or pregnant, or who have an acute infection. The ‘awkward one’ in this classification
system is poliomyelitis, for which both a live attenuated and an inactivated form exist. The Salk form is
inactive (remember the ‘k’ in Salk is also in killed) and the Sabin form is live.

777447225 21
Prepared by: Osama M. Alsurmi

Question 6
Which one of the following statements regarding ciprofloxacin is true?

A. It has poor oral bioavailability


B. It has potent anti-Gram-negative properties, but little or no anti-Gram-positive activity
C. Side-effects involving the gastrointestinal tract and skin are frequent and troublesome
D. It has a small volume of distribution and poor tissue penetration
E. Oral absorption is enhanced by administration with food

Fluoroquinolones such as ciprofloxacin have excellent oral bioavailability, a large volume of distribution
and good intracellular penetration (the drug concentrates in phagocytes, facilitating its distribution).
Although newer quinolones such as moxifloxacin and levofloxacin have enhanced anti-Gram-positive
activity, ciprofloxacin has excellent anti-Gram-negative activity with little or no anti-Gram-positive
activity. Side-effects of this drug are infrequent and generally mild. Its oral bioavailability, as mentioned
above, is excellent, but is impaired by co-administration of food, antacids, ferrous sulphates and
multivitamins.

777447225 22
Prepared by: Osama M. Alsurmi

Question 7
Which one of the following conditions is a notifiable disease in the UK under
the Public Health (Infectious Diseases) Regulations 1988?

A. Chickenpox
B. HIV
C. Scabies
D. Syphilis
E. Measles

Certain diseases that are thought to have important implications for public health are classified as
‘notifiable’, meaning that notification of cases to the local public health authority is compulsory. This is
so that outbreaks of highly contagious conditions can be recognised, and appropriate measures put in
place to control further spread of the condition. In the UK both measles and meningococcal sepsis are
notifiable.

777447225 23
Prepared by: Osama M. Alsurmi

Question 8
Which one of the antibiotics below is correctly paired with its predominant
mode of action?

A. A
B. B
C. C
D. D
E. E

Knowledge of how antibiotics work is important for understanding their spectrum of activity and
mechanisms of resistance. More information is provided in the link below.

777447225 24
Prepared by: Osama M. Alsurmi

Question 9
A previously healthy 14-year-old girl is admitted to the paediatric intensive
care unit with a severe lower respiratory tract infection (LRTI). Serological
investigations subsequently reveal very high titres of Mycoplasma
pneumoniae. Which of the following antibiotics is most effective against this
organism?

A. Flucloxacillin
B. Metronidazole
C. Clarithromycin
D. Co-amoxiclav
E. Ceftriaxone

Clarithromycin is a macrolide antibiotic useful in the treatment of infections caused by Gram-positive,


rickettsial, chlamydial and Mycoplasma infections. Had Legionella, another ‘atypical’ organism, been
identified as the cause (best diagnosed by polymerase chain reaction detection of the Legionella antigen
in urine), then rifampicin would be an appropriate antibiotic to add.

777447225 25
Prepared by: Osama M. Alsurmi

Question 10
Which of the following adverse effects is least likely to occur as a result of
aminoglycoside (e.g. gentamicin) administration?

A. Renal toxicity
B. Cochlear toxicity
C. Neuromuscular blockade following rapid intravenous infusion
D. Hepatotoxicity
E. Nausea and vomiting
Aminoglycoside antibiotics are highly nephrotoxic, an effect potentiated by concomitant administration
of vancomycin, cisplatin, amphotericin B or contrast media. The ototoxicity results from the enhanced
cellular penetration demonstrated by aminoglycosides in the hair cells in the cochlea. Hearing damage
and vertigo tend to be permanent. Aminoglycosides impair neuromuscular transmission and should not be
given to patients with myasthenia gravis. Larger doses given rapidly by the intravenous route may result
in a transient myasthenic syndrome in those with normal neuromuscular function. Gastrointestinal upset
is a relatively common adverse effect but hepatotoxicity does not occur. In order to avoid important
complications, serum levels of the aminoglycosides must be checked on a regular basis, and the dosing
regimen adjusted to ensure that toxic levels are not reached.

777447225 26
Prepared by: Osama M. Alsurmi

Question 11
Which one of the following statements about glycopeptide antibiotics (such
as vancomycin) is incorrect?

A. Rapid infusion may produce an anaphylactoid reaction from histamine release


B. They exhibit good activity against meticillin-resistant Staphylococcus aureus (MRSA)
C. Therapeutic monitoring of drug levels is required
D. Cover against Gram-negative organisms is poor
E. They have good oral bioavailability

No useful oral absorption of glycopeptide antibiotics is achieved, and administration is always by the
intravenous route (except in the treatment ofClostridium difficile diarrhoea). The anaphylactoid reaction
caused by rapid histamine release from mast cells in response to a rapid infusion of vancomycin is known
as ‘red man’ syndrome. Nephrotoxicity is another potential side-effect, the risk of which is greatly
reduced by monitoring of antibiotic serum levels to ensure that toxic concentrations are not reached. No
antibiotic in this class has any significant activity against Gram-negative organisms, but both vancomycin
and teicoplanin are useful against MRSA.

777447225 27
Prepared by: Osama M. Alsurmi

Infectious disease

777447225 28
Prepared by: Osama M. Alsurmi

Question 1
A gap-year student returns from the Far East with a temperature of 39oC.
You take a detailed history of exposures. Which one of the following
exposures is incorrectly paired with the resulting infection?

F. A
G. B
H. C
I. D
J. E

Leptospirosis (Weil’s disease) is caused by infection with the spirochaete, Leptospira interrogans, which
enters humans by direct contact, often through swimming in water contaminated with rat urine.
Mosquitoes may transmit dengue fever, filiariasis and, most importantly, malaria. Hepatitis B may be
contracted from contaminated blood products, unprotected sexual intercourse and the use of intravenous
drugs. Brucellosis and salmonellosis may be contracted through drinking unpasteurised milk.

777447225 29
Prepared by: Osama M. Alsurmi

Question 2
A 24-year-old medical student returns from her elective in Mauritius and
attends the tropical diseases unit with an intensely itchy rash on her foot,
present since just before she came home. She admits to having walked
barefoot a lot while abroad. On examination, there is a serpiginous linear
lesion on the sole of her foot. It is now 7–8 cm in length, and she says that
it has been elongating. What is the most likely diagnosis?

A. Staphyloccus aureus cellulitis


B. Cutaneous larva migrans
C. Scabies
D. Onchocerciasis
E. Strongyloidiasis

The history and description of the rash are classical for cutaneous larva migrans (CLM), which is caused
by the dog hookworm, Ancylostoma caninum. The foot is the most common site, but lesions of CLM
may also be seen on the breasts, buttocks and elbows. Treatment options include albendazole and
ivermectin.

777447225 30
Prepared by: Osama M. Alsurmi

Question 3
Which of the following pairings of childhood exanthems and their
complications is incorrect?

A. A
B. B
C. C
D. D
E. E

Rubella infection in early pregnancy may lead to severe congenital malformation but rarely causes
complications in childhood, with most cases being subclinical. Infection with parvovirus B19 transiently
depresses bone marrow function. This is usually mild but, in individuals with reduced red cell lifespan
(e.g. spherocytosis), it may lead to life-threatening anaemia.

777447225 31
Prepared by: Osama M. Alsurmi

Question 4
A 36-year-old man presents to his general practitioner with a 2-day history
of burning discomfort around the left side of his chest. On examination, you
find a vesicular rash in a band-like distribution around his left side:
Which of the following is the most likely causative organism?

A. Treponema pallidum
B. Herpes simplex virus
C. Varicella zoster virus
D. Staphylococcus aureus
E. Parvovirus B19

The clinical description and rash are typical of shingles, caused by herpes zoster. The virus remains
dormant in the dorsal root ganglion of sensory nerves and can be reactivated at any time.

777447225 32
Prepared by: Osama M. Alsurmi

Question 5
A 16-year-old girl is brought to the hospital accident and emergency
department by her parents, having developed a high fever, headache and
sore throat over the last 12 hours. She tells you that she has just finished
menstruating. On examination, she has a temperature of 38.6°C, blood
pressure of 90/40 mmHg and a generalised erythematous blanching rash.
There are no signs of meningism or petechiae. Which of the following is the
most likely diagnosis?

A. Infectious mononucleosis
B. Staphylococcal toxic shock syndrome
C. Scarlet fever
D. Rubella
E. Rheumatic fever
Staphylococcal toxic shock syndrome is a life-threatening condition that presents with high fever,
systemic upset, a generalised erythematous blanching rash and haemodynamic compromise, before
progressing rapidly to multi-organ failure. It is caused by infection with certain strains of Staphylococcus
aureus that produce a toxin known as toxic shock syndrome toxin 1 (TSST1). A major aetiological factor
is the use of vaginal tampons, particularly when they are not removed in a timely fashion, as Staph.
aureus can grow in and around the tampon with subsequent liberation of TSST1. In the case described
above, the diagnosis of meningococcal sepsis would also have to be considered.

777447225 33
Prepared by: Osama M. Alsurmi

Question 6
A 19-year-old gap-year student returns to the UK after working for 6 weeks
in West Africa on a charity project. Shortly after returning, he presents to
his general practitioner with a 4-day history of intermittent fevers,
tiredness, headache and general malaise. On examination, he is mildly
jaundiced. His heart rate is 110 bpm and temperature is 38.2°C. He has a
palpable spleen and a tender liver edge but no rash or lymphadenopathy.
Which one of the following is the most likely diagnosis?
A. Leishmaniasis
B. Falciparum malaria
C. HIV seroconversion illness
D. Q fever
E. Infectious mononucleosis
A history of intermittent fevers following return from travel in an endemic area should always arouse
suspicion of malaria, the most dangerous form of which is falciparum malaria. Jaundice is common due
to a combination of haemolysis and hepatic dysfunction, anaemia develops rapidly and tender
hepatosplenomegaly is typical. Infectious mononucleosis could potentially present with similar features
but the lack of sore throat, lymphadenopathy or rash and the recent travel history favour malaria. With a
corresponding history of unprotected sexual intercourse, the timing would be right for HIV
seroconversion illness (2–6 weeks after initial infection), which may present non-specifically with fever
and malaise but would not produce anaemia and jaundice. The other two options are highly unlikely.

777447225 34
Prepared by: Osama M. Alsurmi

Question 7
Which of the following conditions is least likely to occur as a consequence of
streptococcal infection?

A. Scarlet fever
B. Endocarditis
C. Rheumatic fever
D. Glomerulonephritis
E. Haemolytic uraemic syndrome

Scarlet fever most often results from uncomplicated streptococcal pharyngitis or tonsillitis. Various
streptococcal species, including the viridans group, may be responsible for endocarditis. In rheumatic
fever, antibodies generated in response to infection with certain strains of group A streptococci cross-
react with cardiac myosin and sacrolemmal membrane proteins, leading to inflammation in the
endocardium, myocardium and pericardium. An immune-mediated reaction following streptococcal
infection may also lead to glomerulonephritis but haemolytic uraemic syndrome is a consequence of
infection with certain strains of Escherichia coli, most notably E. coli O157.

777447225 35
Prepared by: Osama M. Alsurmi

Question 8
All of the following except one are recognised clinical features of
brucellosis. Which is the exception?

A. Pyoderma gangrenosum
B. Uveitis
C. Meningitis
D. Endocarditis
E. Septic arthritis

Brucellae are intracellular organisms that can survive within the body for long periods of time, causing
non-specific symptoms and signs in various organ systems. The acute phase of the illness is marked by
high swinging temperature, rigors and lethargy. Physical findings that are non-specific include
lymphadenopathy and splenomegaly. Brucellosis may cause erythema nodosum but is not associated with
pyoderma gangrenosum.

777447225 36
Prepared by: Osama M. Alsurmi

Question 9
A 45-year-old male who is a keen water-skier is brought to the infectious
diseases unit with a 24-hour history of jaundice, fever and nosebleeds.
There is no history of recent foreign travel. On examination, his temperature
is 39°C, and he is deeply jaundiced and has an enlarged liver. His
conjunctivae are hyperaemic and there is a marked purpuric rash on the
trunk and limbs. Blood results are as follows:
Which of the following is the most likely diagnosis?

A. Listeriosis
B. Hepatitis A
C. Meningococcal sepsis
D. Weil’s disease (leptospirosis)
E. Infectious mononucleosis (Epstein–Barr virus)
This dramatic presentation with fever, haemorrhages, jaundice, purpura, renal impairment and conjuctival
hyperaemia is classical of Weil’s disease, a spirochaetal infection caused by leptospires. These highly
motile organisms are shed in the urine of animal hosts, especially rats, and infection may occur during
prolonged immersion in contaminated water. The disease is therefore more common in freshwater sports
enthusiasts, such as this patient. Meningococcal sepsis must be considered in any patient presenting with
fever and purpura, but this man’s age and the presence of jaundice and hyperaemicconjuctivae make it
less likely. His age also reduces the likelihood of infectious mononucleosis, which does not usually
present in such dramatic fashion. Hepatitis A may present with fever, jaundice and raised transaminases,
but would not account for some of the other features. Listeriosis tends to affect immunocompromised
patients and pregnant women.

777447225 37
Prepared by: Osama M. Alsurmi

Question 10
Which one of the following statements is incorrect with regard
to Escherichia coli?

A. It is the most common cause of travellers’ diarrhoea


B. The incubation period of enterotoxigenic E. coli (ETEC) is 1–2 days
C. Enteropathic E. coli (EPEC) is an important cause of infant diarrhoea
D. Enterohaemorrhagic E. coli (EHEC) may lead to haemolytic uraemic syndrome
E. Entero-invasive E. coli (EIEC) causes symptoms through production of an
enterotoxin

EIEC causes symptoms by direct invasion of the colonic mucosa and presents in a very similar fashion
to Shigella dysentery. Approximately 10–15% of patients with the O157 serotype of EHEC will develop
haemolytic uraemic syndrome, characterised by acute renal failure, and intravascular haemolysis. It
occurs with greatest frequency in the very young and the very old. Management may include dialysis
and/or transfusion. E. coli is also the most common cause of travellers’ diarrhoea.

777447225 38
Prepared by: Osama M. Alsurmi

Question 11
All of the following statements about cytomegalovirus (CMV) are true
except one. Which is the exception?

A. In immunosuppressed patients, infection may cause sight-threatening retinitis


B. In pregnant women, the infection may have neurological sequelae for the fetus
C. In healthy young adults, infection is usually asymptomatic
D. The peak incidence of acquisition is in the sixth decade
E. The virus is shed in saliva, urine and semen

The first peak in virus acquisition is in early childhood, when it is passed readily in communal
environments such as nurseries and school. The second peak occurs in teenage years. In most cases
primary infection is asymptomatic, but carriers often continue to excrete virus for months or years.
Serious consequences of the infection usually occur in the immunosuppressed, in whom it may cause
CMV retinitis, and in pregnant women, when there is a 40% risk of spread to the fetus, 10% of whom
will have serious neurological sequelae.

777447225 39
Prepared by: Osama M. Alsurmi

Question 12
Which one of the following gastrointestinal infections does not cause bloody
diarrhoea?

A. Cholera
B. Enterohaemorrhagic E. coli (EHEC)
C. Shigella
D. Entamoeba histolytica
E. Campylobacter

The bacterium Vibrio cholerae causes acute watery diarrhoea. The stool is typically ‘rice water’ in nature,
consisting of clear fluid with flecks of mucus. This contrasts with the other organisms listed, which may
directly invade the mucosa of the small bowel, or produce cytotoxins that damage and ulcerate the
mucosa with inflammation, producing bloody diarrhoea.

777447225 40
Prepared by: Osama M. Alsurmi

Question 13
Which one of the following statements regarding visceral leishmaniasis
is incorrect?

A. It is transmitted from human to human by the sandfly


B. Both splenomegaly and hepatomegaly are common clinical features
C. Dermatological manifestations include black discoloration of the skin
D. Blood tests typically reveal thrombocytosis and neutrophilia
E. The treatment of choice is pentavalent antimony

Characteristically, pancytopenia is found on full blood count, rather than thrombocytosis and
neutrophilia.

777447225 41
Prepared by: Osama M. Alsurmi

Question 14
Which one of the following statements is correct regarding the correction of
fluid and electrolyte balance in the patient with acute diarrhoea?
A. 48 hours of diarrhoea in the average adult will result in a fluid deficit of half a litre
B. Intravenous fluid replacement is preferable to oral rehydration for most patients with acute diarrhoea
C. Oral rehydration solution (ORS) contains carbohydrate, sodium, potassium and chloride
D. Prolonged bouts of diarrhoea result in the development of a metabolic alkalosis
E. Infants with gastroenteritis should not be breastfed while they have diarrhoea
Replacement of fluid losses is by far the most important aspect of the management of acute
gastroenteritis. The average adult with 48 hours of moderate diarrhoea will be 1–2 L depleted. Adequate
rehydration requires replacement of this deficit and on-going diarrhoeal losses in addition to normal daily
fluid requirement. While intravenous replacement may be required for those who cannot keep up with
losses by oral intake alone (e.g. in persistent vomiting), the vast majority of cases can be managed with
appropriate oral rehydration. ORS contains both electrolytes and a source of carbohydrate to allow
absorption to take place. Prolonged bouts of diarrhoea will result in a metabolic acidosis, due to the loss
of bicarbonate-rich secretions from the gastrointestinal tract distal to the stomach. Infants with
gastroenteritis should definitely continue to breastfeed, as milk is sterile and nutritious, and contains the
correct balance of electrolytes.

777447225 42
Prepared by: Osama M. Alsurmi

Question 15
A sales executive, recently returned to the UK from Ukraine, is referred to
the local infectious diseases unit by his general practitioner with a sore
throat of unusual appearance. On arrival, he is tachycardic with a heart rate
of 110 bpm, and has a mild pyrexia of 37.7°C. There is a slight blood-stained
discharge at the nasal orifices. His neck is visibly swollen. Examination of
the mouth reveals a greyish-green membrane overlying the tonsils, which
does not come off with gentle scraping with the tongue depressor. What is
the likely diagnosis?
A. Streptococcal throat infection
B. Diphtheria
C. Oral candidiasis
D. Mumps
E. Tonsillar carcinoma
Although diphtheria was eradicated from the developed world in the mid-20th century, it remains an
important source of illness in the developing world, and recent years have seen the occurrence of outbreaks
in South-east Asia and parts of the former USSR. The ‘bull neck’ and ‘wash-leather’ grey–green membrane
covering the tonsils are characteristic of the acute presentation. Treatment is threefold, including
administration of diphtheria antitoxin, antibiotics (penicillin or erythromycin) and isolation. A streptococcal
sore throat will characteristically look red and raw; oral candidiasis is whitish in appearance, and has a
predilection for the tongue rather than the tonsils; mumps usually affects a younger age group and may
cause a swollen neck, but not the other findings; tonsillar cancer is exceedingly rare and does not really fit
with this acute presentation.

777447225 43
Prepared by: Osama M. Alsurmi

Question 16
A 45-year-old Indian man visiting his family in the UK is brought to the
infectious diseases unit by his son, who is concerned about the appearance
of what he terms a ‘rash’ on his father’s back. On examination, there are
three well-defined hypopigmented macules that are completely anaesthetic.
You note that both the patient’s median and ulnar nerves are palpably
thickened bilaterally, with variable sensory loss distal to the wrists in both
upper limbs. Which of the following is the most likely underlying diagnosis?

A. Toxoplasmosis
B. Botulism
C. Cutaneous leishmaniasis
D. Leprosy
E. Tuberculosis

A history of skin lesions and thickened peripheral nerves should lead to strong suspicion of leprosy.
About 70% of the world’s leprosy patients live in India, so this gentleman’s ethnic origin is also a clue.

777447225 44
Prepared by: Osama M. Alsurmi

Question 17
With regard to the patient in the previous question, which one of the
following investigations will confirm the diagnosis of leprosy?

A. Leprosy serology
B. Polymerase chain reaction (PCR) of serum for Mycobacterium leprae DNA
C. Microscopy of slit skin smears
D. Nasal swab
E. Blood culture

Although the diagnosis of leprosy is largely clinical, based on the presence of cardinal features, the
finding of acid-fast bacilli in slit skin is confirmatory and also useful for monitoring response to
treatment. M. leprae still cannot be grown in vitro and neither serology nor PCR testing for leprosy DNA
is sensitive enough for diagnostic purposes.

777447225 45
Prepared by: Osama M. Alsurmi

Question 18
A 14-year-old boy, living in London, is brought to the hospital accident and
emergency department by his mother with a 6-day history of malaise, fever
and sore throat. He gives no history of recent foreign travel. His
temperature is 38°C. On examination, you note that he has an inflamed
pharynx, cervical lymphadenopathy, a tender enlarged liver and a palpable
spleen. What is the most likely diagnosis?

A. Infectious mononucleosis
B. Malaria
C. Influenza
D. Mumps
E. Listeriosis

The symptoms, clinical findings, lack of travel history and age group all point towards a diagnosis of
infectious mononucleosis, caused by the Epstein–Barr virus.

777447225 46
Prepared by: Osama M. Alsurmi

Question 19
Which of the following antimicrobial agents is commonly used in the
treatment of leprosy?

A. Clindamycin
B. Chloroquine
C. Chloramphenicol
D. Rifampicin
E. Penicillin V

Rifampicin is a potent bactericidal for Mycobacterium leprae but should always be given in combination
with other antileprotics. Multidrug treatment (MDT) is crucial to treatment success because of the
emergence of resistant strains of the bacillus. More than 12 million patients have now been successfully
treated with MDT.

777447225 47
Prepared by: Osama M. Alsurmi

Question 20
All of the following except one are potential complications
of falciparum malaria. Which is the exception?

A. Convulsions
B. Acute renal failure
C. Nephrotic syndrome
D. Hypoglycaemia
E. Metabolic acidosis

Infection with Plasmodium malariae but not Plasmodium falciparum may cause glomerulonephritis and
nephrotic syndrome in children. Renal complications of falciparummalaria include acute renal failure
secondary to acute tubular necrosis, occasionally with haemoglobinuria (‘blackwater fever’).

777447225 48
Prepared by: Osama M. Alsurmi

Question 21
Which of the following antibiotics is used as first-line therapy for
symptomatic infection withClostridium difficile?

A. Co-amoxiclav
B. Metronidazole
C. Cefuroxime
D. Clarithromycin
E. Clindamycin

A 10-day course of oral metronidazole is first-line therapy for established C. difficile infection; oral
vancomycin is used if there is no response. Almost all antibiotics predispose to C. difficileinfection,
particularly broad-spectrum penicillins (e.g. co-amoxiclav), second/third-generation cephalosporins (e.g.
cefuroxime) and clindamycin.

777447225 49
Prepared by: Osama M. Alsurmi

Question 22
Which one of the following investigations is diagnostic for malaria?

A. Thin blood film


B. Full blood count
C. Blood culture
D. Serology for P. falciparum
E. Coagulation screen

A thin blood film allows identification of the species of parasite as well as estimation of parasitic load.
Full blood and coagulation screen will commonly reveal abnormalities but are not diagnostic. Culture and
serological tests play no role in the diagnosis.

777447225 50
Prepared by: Osama M. Alsurmi

Question 23
A 19-year-old woman is referred by her general practitioner to the local
dermatology department, 3 months after returning from a trip to Central
America. She has an ulcer on the lateral aspect of her calf, which has failed
to heal with a 2-week course of flucloxacillin. It began 1 month previously
as a raised red ‘lump’, then increased in size to about 6 cm, before
ulcerating and developing a crust. Swabs of the ulcer have grown skin
commensals only but a biopsy shows numerous parasites. What is the most
likely diagnosis?

A. Toxoplasmosis
B. Cutaneous leishmaniasis
C. Babesiosis
D. Plasmodium vivax malaria
E. Trypanosomiasis
The history of the lesion, recent travel to an endemic area and biopsy findings of parasites in the dermis
all make cutaneous leishmaniasis (CL) by far the most likely diagnosis. The incubation period is typically
2–3 months. Although all of the above are protozoal infections, only CL gives a cutaneous ulcer as
described.

777447225 51
Prepared by: Osama M. Alsurmi

Question 24
Infection with which one of the following organisms is most commonly
associated with cardiac complications, including dilated cardiomyopathy and
conduction defects?

A. Toxoplasma gondii
B. Trypanosoma cruzi
C. Leishmania donovani
D. Ancylostoma braziliense
E. Cytomegalovirus

American trypanosomiasis (Chagas’ disease) commonly occurs in South and Central America, and is
transmitted to humans by contact with the faeces of the reduviid bug. Between 10% and 30% of cases
with chronic infection develop a cardiomyopathy characterised by cardiac dilatation, varying degrees of
atrioventricular block, arrhythmias and sudden death.

777447225 52
Prepared by: Osama M. Alsurmi

Question 25
With regard to toxoplasmosis, which one of the following statements
is incorrect?

A. Domestic cats are important hosts in the life cycle of Toxoplasma gondii
B. Infection is asymptomatic in the majority of individuals
C. Congenital toxoplasmosis is characterised by retinochoroiditis, microcephaly and hydrocephalus
D. Serological tests are available for the detection of past and present infection withToxoplasma gondii
E. The most common presenting features of acquired toxoplasmosis are fever, photophobia and
a non-specific rash

The most common clinical presentation of toxoplasmosis is with painless enlargement of lymph nodes,
particularly cervical nodes. Photophobia and a rash do not normally feature. The risks of transmission to
the fetus are greatest in the third trimester; however, the overall incidence and severity of congenital
disease are greatest in the first trimester.

777447225 53
Prepared by: Osama M. Alsurmi

Question 26
Which one of the following statements regarding amoebiasis is correct?

A. The parasite typically invades the small bowel


B. The incubation period of the Entamoeba histolytica cyst is 48 hours
C. Clinically, the disease manifests as diarrhoea alternating with constipation, with the
presence of blood and mucus
D. Diagnosis is made on stool culture
E. The infection is responsive to penicillin antibiotics

Amoebiasis is a form of dysentery, which is spread to humans by ingestion of cysts of E. histolytica in


water or uncooked food. Vegetative trophozoite forms emerge from cysts in the large bowel and invade
the mucous membrane. The incubation period ranges from 2 weeks to many years. Diagnosis is made by
direct microscopy of stools, looking for motile trophozoites. Intestinal amoebiasis responds rapidly to
oral metronidazole.

777447225 54
Prepared by: Osama M. Alsurmi

Question 27
A 53-year-old man, recently returned to the UK from an extended stay in
Indonesia, presents with a tender, erythematous right leg and thigh. On
examination, his right leg is swollen, warm, painful and red along the length
of its medial aspect. There is marked oedema. He also has an acutely tender
right testicle and swelling of the right inguinal lymph nodes. A full blood
count reveals eosinophilia. Which one of the following is the most likely
diagnosis?

A. Acute streptococcal cellulitis


B. Filariasis
C. Cutaneous larva migrans infection
D. Tinea pedis
E. Histoplasmosis
Filarial worms are tissue-dwelling nematodes; toxins released by the adult worm after dissemination in
the human blood stream cause dilatation of the lymphatic vessels, leading to lymphatic dysfunction and
the chronic limb manifestation of elephantiasis. In the acute phase, the presentation is as described above,
with fever, erythema and tenderness along the course of the inflamed lymphatic vessels and epididymo-
orchitis. The treatment of choice is diethylcarbamazine (DEC).

777447225 55
Prepared by: Osama M. Alsurmi

Question 28
The following are all recognised sequelae of chronic infection with
schistosomiasis except one. Which is the exception?

A. Hydronephrosis
B. Oesophageal varices
C. Splenomegaly
D. Urticaria
E. Recurrent urinary tract infection

Urticaria is a manifestation of acute schistosomal infection. Schistosoma haematobiumcharacteristically


affects the urinary tract, causing the chronic complications of recurrent urinary tract infections, bladder or
ureteric stone formation, and hydronephrosis. The eggs of S. mansoni and S. japonicum are deposited in
the liver, where they may lead to portal hypertension and consequent development of splenomegaly,
progressive ascites and oesophageal varices.

777447225 56
Prepared by: Osama M. Alsurmi

Question 29
A traveller, returning from East Africa to the UK with a fever of 38.7oC, has
a full blood count performed as part of his initial work-up. This reveals an
eosinophilia (eosinophil count 0.9 × 109). Which one of the following
diagnoses is least consistent with this finding?

A. Schistosomiasis
B. Strongyloidiasis
C. Ascariasis
D. Tuberculosis
E. Filariasis

Eosinophils play an important role in combatting helminths, phagocytosing immune complexes and
modulating type 1 hypersensitivity reactions. Eosinophilia is therefore most commonly seen with
parasitic infections and allergy. Tuberculosis causes a monocytosis.

777447225 57
Prepared by: Osama M. Alsurmi

Question 30
A 24-year-old student presents to the dermatology clinic with an expanding
red macule on her left calf; this appeared 2 weeks previously, shortly after
returning from holiday in the north-east USA. She reports that the lesion has
been progressively increasing in size since it appeared. On examination, the
lesion now measures approximately 15 cm across and appears to be
clearing from the centre outwards. Which of the following is the likely
diagnosis of her rash?

A. Necrobiosis lipoidica
B. Erythema nodosum
C. Pyoderma gangrenosum
D. Cellulitis
E. Erythema migrans
Erythema (chronicum) migrans is the rash associated with Lyme disease, caused by infection with
spirochaetes of the genus Borrelia. The main vector is the tick Ixodes, whose bites transmit the disease to
humans. The early disease is localised, manifesting as a skin reaction around the site of the tick bite. The
infection may then disseminate to produce widespread systemic effects such as arthralgia, meningitis,
cranial nerve palsies and peripheral neuropathy. Late advanced disease produces arthritis, encephalopathy
and acrodermatitis chronica atrophicans.

777447225 58
Prepared by: Osama M. Alsurmi

Question 31
A 19-year-old male attends his general practitioner’s surgery 3 days after
returning from back-packing in South-east Asia. He complains of headache,
generalised aches and pains, painful eye movements and a rash. He recalls a
number of mosquito bites in the week prior to returning home but appears
to have taken appropriate antimalarial prophylaxis throughout the trip and
since his return. On examination, he has lymphadenopathy, a temperature
of 38.2oC, and a maculopapular morbilliform rash on his trunk. Which one of
the following diagnoses is most likely?
A. Salmonella infection
B. Schistosomiasis
C. Leishmaniasis
D. Dengue
E. Strongyloidiasis

Despite the apparent compliance with chemoprophylaxis, it would be vital to consider the diagnosis of
malaria in this and any other patient recently returned from an endemic area who has a fever. Such patients
should have urgent thick and thin blood films sent to look for thePlasmodium falciparum parasite.
Nevertheless, by far the more likely diagnosis in this scenario would be dengue fever, considering the
recent travel history, mosquito bites and the symptoms described. The mosquito, Aedes aegypti, is the
principal vector, and the incubation period is 2–7 days, with the rash appearing between days 3 and 5 of the
symptomatic infection. A more serious form of the infection, dengue haemorrhagic fever, can also occur,
particularly in children in South-east Asia.

777447225 59
Prepared by: Osama M. Alsurmi

Question 32
Regarding measles, which one of the following statements is correct?

A. The incubation period is 5 days to onset of rash


B. Koplik’s spots on the retina are pathognomonic
C. Antibiotics should be given to prevent the development of bacterial pneumonia
D. Infants are protected for the first 6 months of life by passive immunity from maternal
antibodies
E. Children with the disease should be offered vaccination as soon as they recover

The incubation period in measles is 14 days. Koplik’s spots are pathognomonic of measles but appear on
the internal buccal mucosa not the retina. Bacterial pneumonia complicates the disease in around 4% of
cases and accounts for many measles-related deaths, but antibiotics should not be given unless there is
clinical evidence of superinfection (see evidence-based medicine box in link). Vaccination after recovery
is pointless, as survival of infection confers life-long immunity.

777447225 60
Prepared by: Osama M. Alsurmi

Question 33
Which of the five senses is most commonly affected by infection
with Onchocerca volvulus(onchocerciasis)?

A. Vision
B. Taste
C. Hearing
D. Touch
E. Smell

Onchocerciasis, otherwise known as river blindness, is the result of infection by the filarialOnchocerca
volvulus, transmitted by flies of the genus Simulium. It is endemic in sub-Saharan Africa, where it leaves
almost half a million people with visual impairment every year. Eye disease is heralded by itching,
lacrimation and conjunctival injection. ‘Snowflake’ deposits are classically seen at the edge of the cornea.
The treatment of choice is ivermectin, which is also in widespread use as a prophylactic agent.

777447225 61
Prepared by: Osama M. Alsurmi

HIV infections & AIDS

777447225 62
Prepared by: Osama M. Alsurmi

Question 1
Which of the following best describes the human immunodeficiency virus
(HIV)?

A. Single-stranded RNA virus


B. Double-stranded RNA virus
C. Single-stranded DNA virus
D. Double-stranded DNA virus
E. Prion

HIV is a single-stranded RNA virus. It consists of a protein core, surrounded by a lipid membrane that is
lined by a matrix protein studded with glycoprotein. The virus requires a host cell (a CD4-positive cell) in
order to replicate, and to integrate its own genetic material into that of the host.

777447225 63
Prepared by: Osama M. Alsurmi

Question 2
Which one of the following options best characterises the virological and
immunological progression of HIV infection in the first 4–8 weeks following
exposure?

A. CD4 count rises, viral load falls


B. CD4 count falls, viral load rises
C. Both CD4 count and viral load rise
D. Both CD4 count and viral load fall
E. CD4 count remains stable, viral load rises

Primary infection with HIV is symptomatic in more than 50% of cases, presenting with symptoms
suggestive of a glandular fever-like illness. This coincides with a surge in the plasma HIV-RNA levels to
> 1 million copies/mL, and a fall in the CD4 count.

777447225 64
Prepared by: Osama M. Alsurmi

Question 3
Which one of the following is not an AIDS-defining disease?

A. Pneumocystis pneumonia
B. Kaposi’s sarcoma
C. Cytomegalovirus retinitis
D. Oropharyngeal candidiasis
E. Cerebral toxoplasmosis

The acquired immunodeficiency syndrome (AIDS) is defined by the development of specified


opportunistic infections, tumours etc. (see link below) in a patient with HIV infection. These tend to
become manifest with varying degrees of immunosuppression, so that there is a correlation between the
CD4 count and the specific diseases likely to occur. Oesophageal, but not oropharyngeal, candidiasis is
considered an AIDS-defining illness.

777447225 65
Prepared by: Osama M. Alsurmi

Question 4
Which one of the following statements regarding Cryptosporidium infection
isincorrect?

A. Patients typically present with profuse, bloody diarrhoea


B. It usually occurs when the CD4 count is <100 cells/mm3
C. Definitive diagnosis is made on stool microscopy
D. Fever is typically absent
E. Cryptosporidium has multiple animal hosts

One of the cardinal features of Cryptosporidium infection is that the diarrhoea is watery and cholera-like,
as opposed to bloody. When prolonged, it is an AIDS-defining illness that typically occurs when the CD4
count falls below 100 cells/mm3. HIV-infected patients should be advised to wash their hands after
potential contact with human or animal faeces, and to avoid drinking untreated water.

777447225 66
Prepared by: Osama M. Alsurmi

Question 5
Which one of the following options is a characteristic feature
of Pneumocystis jirovecii (formerly carinii) pneumonia?

A. Acute (<7 days) history of respiratory complaints


B. A cough productive of copious amounts of mucopurulent sputum
C. Marked oxygen desaturation on exercise
D. Pleural effusions
E. Leukocytosis

Pneumocystis jirovecii (carinii) pneumonia is a subacute AIDS-defining illness that affects the respiratory
tract, causing a dry cough, pyrexia and breathlessness. The chest radiograph typically shows a bilateral
‘ground glass’ interstitial infiltrate. Exercise-related desaturation is a common feature. Diagnosis is based
on cytology of induced sputum samples or bronchoalveolar lavage, as the organism cannot be cultured.
First-line treatment is high-dose co-trimoxazole and steroids. Pneumocystis infection typically occurs
when the CD4 count falls below 200 cells/mm3.

777447225 67
Prepared by: Osama M. Alsurmi

Question 6
Which one of the following statements regarding cryptococcal meningitis
is true?
A. The organism is identified on cerebrospinal fluid (CSF) microscopy with Ziehl–Neelsen staining
B. The majority of affected patients have a CD4 count > 200 cells/mm3
C. Raised intracranial pressure is best treated with therapeutic lumbar punctures
D. Neck stiffness is invariably present
E. Penicillin is the treatment of choice

Cryptococcus may be identified on microscopy of CSF that has been stained with India ink, but the most
sensitive test is the detection of cryptococcal antigen in CSF. It is the most common cause of meningitis
in late-stage HIV infection, and tends to occur only once the CD4 count has fallen below 100 cells/mm3.
The typical signs of meningism are often absent, and the presenting features may be non-specific, with
headache, fever, vomiting and mild confusion. Deafness and blindness may result from prolonged
elevation of intracranial pressure, which is due to a communicating hydrocephalus and should be treated
with repeat therapeutic lumbar punctures. Cryptococcus neoformans is a budding encapsulated yeast, and
therefore penicillin-based antibiotic regimens are ineffective. Amphotericin B given intravenously
together with flucytosine is the initial treatment of choice. After 14 days, fluconazole is given until
immunity has recovered on antiretroviral therapy.

777447225 68
Prepared by: Osama M. Alsurmi

Question 7
A 27-year-old HIV-positive patient with a CD4 count of 20
cells/mm3 presents to the hospital accident and emergency department
with visual difficulties. He reports a 6-week history of flashing lights and
dark ‘clouds’ floating in front of his eyes. In the past fortnight he has
experienced problems with driving, narrowly escaping a collision with a
cyclist whom he did not see on the road beside him. Fundoscopy reveals
haemorrhagic exudates near the retinal vessels, with a clear vitreous. What
is the likeliest diagnosis?

A. Ocular toxoplasmosis
B. Herpes simplex keratitis
C. Anterior uveitis
D. Cytomegalovirus (CMV) retinitis
E. HIV retinopathy
The symptoms given in the history, along with the time-scale of the visual loss and the physical findings,
are all suggestive of CMV retinitis. Although the incidence has plummeted since the introduction of
antiretroviral therapy (ART), the infection is sight-threatening, particularly if the macula is affected. It
tends to occur when the CD4 count falls below 50 cells/mm3. Treatment involves the initiation of antivirals
for CMV (e.g. ganciclovir, cidofovir) and ART, if not already instituted. Visual loss may result if treatment
is not started promptly.

777447225 69
Prepared by: Osama M. Alsurmi

Question 8
A 37-year-old homosexual male who is HIV-positive (CD4 count 300
cells/mm3) is found, on a routine visit to the infectious diseases clinic, to
have purplish-blue papules across the bridge of his nose; these have
appeared in the last 4 months. He is otherwise asymptomatic. Which one of
the following diagnoses is most likely?

A. Seborrhoeic dermatitis
B. Disseminated cutaneous herpes simplex
C. Molluscum contagiosum
D. HIV-associated lymphoma
E. Kaposi’s sarcoma

Options A–C form part of the differential diagnosis of HIV-related skin disease but do not fit well with
this presentation. T-cell lymphomas involve the skin but these are not associated with HIV; the AIDS-
defining lymphoma is of B-cell origin. Kaposi’s sarcoma, an AIDS-defining disease, is a tumour that
affects cutaneous, oral and visceral sites. It may follow an indolent or a fulminant course, with visceral
involvement often following cutaneous disease. The pattern of disease mirrors the CD4 count, with
accelerated disease more common in those with low CD4 counts.

777447225 70
Prepared by: Osama M. Alsurmi

Question 9
Which one of the following opportunistic infections is incorrectly matched
with its appropriate prophylactic agent?

A. A
B. B
C. C
D. D
E. E

Appropriate prophylaxis for toxoplasmosis would be co-trimoxazole. The decision to institute


prophylaxis is generally dependent on CD4 count (except tuberculosis), in that prophylaxis will be started
when the CD4 count falls to a level at which the patient is deemed to have much greater susceptibility to
that particular infection. For example, for Pneumocystis jirovecii pneumonia, the indication to commence
co-trimoxazole is when the CD4 count falls below 200 cells/mm3. Clinical staging is also an indication to
commence co-trimoxazole prophylaxis, independent of the CD4 count.

777447225 71
Prepared by: Osama M. Alsurmi

Question 10
Which of the following exposures has the highest transmission risk for HIV?

A. Genital mucous membrane contact


B. Intravenous drug use using a contaminated needle
C. Being born to a mother who is HIV-positive but not on antiretroviral therapy (ART)
D. Receptive anal intercourse
E. Receipt of blood transfusion from an HIV-infected donor

Receipt of infected blood transfusion carries a 90% risk of transmission to the recipient. The other
transmission risks are:

777447225 72
Prepared by: Osama M. Alsurmi

Sexually transmitted infections

777447225 73
Prepared by: Osama M. Alsurmi

Question 1
A 22-year-old heterosexual male presents to the STI clinic with a 2-week
history of purulent urethral discharge and dysuria. He has been sexually
active in the past month with two new partners. Gram-stained smear of
urethral exudate shows Gram-negative diplococci, along with prominent
polymorphonuclear cells. What is the most likely cause of the man’s urethral
discharge?

A. Non-gonococcal urethritis
B. Syphilis
C. Genital warts
D. Gonorrhoea
E. Staphylococcal infection

The presence of Gram-negative diplococci on a smear of urethral exudate makes gonococcal urethritis the
most likely diagnosis; culture may subsequently confirm the organism. Note that if a patient has been
sexually active with two new partners in the past month, then the likelihood of an STI is increased.

777447225 74
Prepared by: Osama M. Alsurmi

Question 2
Which of the following genotypes of human papillomavirus (HPV) are
associated with dysplastic conditions and cancers of the genital tract?

A. HPV 6 and 11
B. HPV 16 and 18
C. HPV 10 and 15
D. HPV 6 only
E. HPV 10 and 13

HPV 16 and 18 have been linked to neoplasia of the genital tract. HPV 6 and 11 are the relatively
benign strains that cause anogenital warts.

777447225 75
Prepared by: Osama M. Alsurmi

Question 3
A 34-year-old female is referred to the local gynaecology outpatient
department with a 2-year history of intermittent pelvic pain. In her history,
she also describes dyspareunia and occasional postcoital bleeding. She and
her husband have been trying to conceive without success for the past 18
months. Examination of the external genitalia is unremarkable. On per
vaginam examination, you see an inflamed cervix, which is partly covered
by a mucopurulent discharge. Which of the following diagnoses is most
likely?

A. Syphilis
B. Herpes simplex type 1 infection
C. Herpes simplex type 2 infection
D. Trichomoniasis
E. Chlamydial infection
The most commonly diagnosed curable STI in the UK is chlamydia, with approximately 100 000 new cases
detected each year. Chlamydia is far more frequently diagnosed than the next most common curable STI,
gonorrhoea, of which 22 000 cases are reported each year. Chlamydia is usually asymptomatic, but may
cause symptoms of pelvic inflammatory disease such as those described above. In either scenario, it may
eventually cause damage to the Fallopian tubes, and subsequently infertility or ectopic pregnancy. Other
rare complications in either sex include conjunctivitis and Reiter’s syndrome.

777447225 76
Prepared by: Osama M. Alsurmi

Question 4
A 35-year-old female is referred to Gynaecology triage with a 7-day history
of a profuse, foul-smelling vaginal discharge. She is very distressed by this,
as the quantities produced are necessitating the use of sanitary towels. On
speculum examination, the external genitalia are entirely normal but the
vagina itself is slightly inflamed. High and low vaginal swabs are taken, as
well as a wet film of some of the discharge, which is sent immediately for
microscopy. Motile flagellate organisms are seen in large numbers. What is
the diagnosis?

A. Trichomoniasis
B. Bacterial vaginosis
C. Herpes simplex type 2 infection
D. Chlamydial infection
E. Gonorrhoea

Motile flagellate protozoa seen on a wet mount of vaginal material are diagnostic of trichomoniasis. The
organism involved is Trichomonas vaginalis, and the treatment of choice is metronidazole.

777447225 77
Prepared by: Osama M. Alsurmi

Question 5
A 17-year-old female reluctantly attends her general practitioner’s surgery
to discuss the problem of a 2-month history of a whitish vaginal discharge
and vulval pruritus. She is not sexually active and her only past medical
history is of acne, for which she takes regular minocycline. On examination,
her external genitalia are inflamed and traces of a curd-like discharge can
be seen at the vaginal orifice. What is the most likely diagnosis?

A. Human papillomavirus infection


B. Vaginal candidiasis
C. Chlamydial infection
D. Toxic shock syndrome (TSS)
E. Gonorrhoea
The clinical picture best fits candidiasis. Options A, C and E would be unlikely in a woman of this age who
is not sexually active, and the history would not fit with any of these. Although TSS could occur in this
patient, it would present in an entirely different way. A further clue to the diagnosis lies in the fact that the
patient is on long-term antibiotic therapy for acne; this would, of course, predispose her to candidal
infection, as the antibiotics disrupt the natural flora of the female genital tract, allowing overgrowth of
pathogens such as Candida albicans.

777447225 78
Prepared by: Osama M. Alsurmi

Question 6
A 23-year-old male presents to his general practitioner with a 3-day history
of a painful rash on his penis. This was preceded by 2 days of ‘flu-like
symptoms’. He has been sexually active with a new partner for the last 6
weeks but does not use condoms. On examination, he has a cluster of small
vesicles on the shaft of his penis. Which is the most appropriate next
investigation?

A. Dark-ground microscopy for Treponema pallidum


B. Urethral swab
C. HIV test
D. Swab of the contents of a vesicle
E. Urine microscopy and culture

The most likely diagnosis in this case is genital herpes, which is diagnosed by sending swabs of vesicular
fluid or ulcers for polymerase chain reaction examination for detection of herpes simplex virus (HSV)
DNA. Although HSV-2 is classically associated with anogenital herpes, HSV-1 is found in about 50% of
cases in the UK, as well as being the most common strain found in orolabial herpes.

777447225 79
Prepared by: Osama M. Alsurmi

Question 7
Which one of the following is not a feature of secondary syphilis?

A. Condylomata lata
B. Meningitis
C. Generalised lymphadenopathy
D. A maculo-papular rash on the trunk and limbs
E. Syphilitic chancre

A chancre is an indurated ulcer, which is the primary lesion of syphilis; it occurs in the first days of
symptomatic infection. It begins as a dull red macule, which becomes papular and eventually ulcerates.
Draining lymph nodes may also become enlarged but are painless. The chancre is usually located on the
penis in men or on the labia in women. Chancres may also occur on the vaginal wall and on the cervix. In
around 10% of cases, extra-genital sites such as fingers, lips, tongue and nipple may be involved.
Secondary syphilis occurs 6–8 weeks after the chancre, when the treponemes have disseminated to produce
multi-system disease.

777447225 80
Prepared by: Osama M. Alsurmi

Endocrine

777447225 81
Prepared by: Osama M. Alsurmi

Question 1
A 38-year-old woman is referred to the medical outpatient clinic with a 6-
month history of heat intolerance, palpitations and weight loss despite a
normal appetite. Examination reveals a fine tremor and sinus tachycardia
but is otherwise unremarkable. Blood tests show the following results:
TSH receptor antibodies are detected in her serum. What is the most likely
diagnosis?

A. Toxic multinodular goitre


B. Toxic thyroid adenoma
C. Graves’ thyrotoxicosis
D. Secondary thyrotoxicosis due to a pituitary tumour
E. Factitious thyrotoxicosis

The raised T4 with undetectable TSH levels indicates primary thyrotoxicosis. The most common cause of
thyrotoxicosis is the autoimmune condition, Graves’ disease. TSH receptor antibodies are very rare in
patients without autoimmune thyroid disease and their presence therefore confirms the diagnosis. Note,
however, that 5–20% of patients with Graves’ disease are negative for these antibodies; had the result been
negative rather than positive, this would not have excluded the diagnosis of Graves’ disease and further
tests (thyroid scintigraphy) might have been required.

777447225 82
Prepared by: Osama M. Alsurmi

Question 2
A 59-year-old man with clinical and biochemical evidence of thyrotoxicosis
undergoes 99mtechnetium scanning (scintigraphy) of the thyroid. This
reveals a well-defined area of increased isotope uptake with no uptake
through the rest of the gland. What is the most likely diagnosis?

A. Thyroiditis
B. Papillary thyroid carcinoma
C. Toxic multinodular goitre
D. Medullary carcinoma of the thyroid
E. Toxic thyroid adenoma
In toxic adenoma there is increased isotope uptake by the adenoma. However, excessive thyroxine
production by the adenoma inhibits pituitary thyroid-stimulating hormone (TSH) secretion so that the
remainder of the thyroid gland is suppressed, resulting in negligible isotope uptake. In toxic multinodular
goitre there is patchy uptake by the hyperfunctioning nodules. Thyroiditis is associated with low isotope
uptake by the thyroid, as the follicular cells are not functioning normally during a temporary inflammation
of the gland. (The excess thyroid hormone is released from stores within the follicle, rather than synthesised
de novo.) Thyroid malignancies are not usually associated with derangements of thyroid function and are
visible as ‘cold’ (low-uptake) areas on a scintigraphy scan. Medullary carcinoma arises from the
parafollicular C cells that produce calcitonin. A very useful guide to establishing the diagnosis in
thyrotoxicosis is provided in the link below.

777447225 83
Prepared by: Osama M. Alsurmi

Question 3
Which of the following ocular signs of thyrotoxicosis is not specific to
Graves’ disease?

A. Exophthalmos
B. Lid lag
C. Periorbital oedema
D. Chemosis
E. Ophthalmoplegia

Lid retraction (visible sclera above the iris) and lid lag (delay in downward movement of the upper eyelid
during downgaze) can occur with all causes of thyrotoxicosis. They result from the effect of increased
sympathetic activity on the levatorpalpebraesuperioris muscles. The other listed ocular features are only
seen in Graves’ disease, although their presence is unrelated to the severity of the thyrotoxicosis.
Ophthalmoplegia and diplopia result from swelling and eventual fibrosis of the extraocular muscles. Other
features specific to Graves’ are pretibial myxoedema and thyroid acropachy (finger clubbing).

777447225 84
Prepared by: Osama M. Alsurmi

Question 4
Which one of the following types of autoantibody is most strongly
associated with Graves’ disease?

A. Anti-thyroid peroxidase antibodies


B. Anti-thyroglobulin antibodies
C. Antinuclear antibodies (ANA)
D. Thyroid-stimulating hormone receptor antibodies (TRAb)
E. Antineutrophil cytoplasmic antibodies (ANCA)

Stimulating TRAbs are responsible for stimulating the thyroid-stimulating hormone receptors and causing
thyrotoxicosis. Occasionally, blocking TRAb antibodies cause episodes of hypothyroidism in patients with
autoimmune thyroid disease. Routine assays cannot distinguish stimulating from blocking antibodies, but
TRAbs are present in up to 95% of patients with Graves’ disease and are rare in patients with other forms
of thyroid disease.

777447225 85
Prepared by: Osama M. Alsurmi

Question 5
All of the following except one are recognised causes of hypothyroidism.
Which is the exception?

A. Hashimoto’s thyroiditis
B. Post-partum thyroiditis
C. Graves’ disease
D. Primary hyperparathyroidism
E. Radioactive iodine treatment for hyperthyroidism

The most common causes of hypothyroidism in the UK are autoimmune disease (including Hashimoto’s
thyroiditis) and thyroid failure following thyroidectomy or radio-iodine therapy. Transient thyroiditis (e.g.
de Quervain’s thyroiditis, post-partum thyroiditis) is often associated with an initial thyrotoxic phase,
followed by transient hypothyroidism. Although Graves’ disease usually causes hyperthyroidism, the
thyroid-stimulating hormone receptor antibodies can sometimes have a blocking effect rather than a
stimulatory one, resulting in hypothyroidism. Primary hyperparathyroidism does not cause hypothyroidism.

777447225 86
Prepared by: Osama M. Alsurmi

Question 6
A 52-year-old woman attends her general practitioner with a 1-year history
of fatigue, cold intolerance, weight gain and pain in her left hand at night.
Blood tests reveal the following:
The most appropriate initial management would be:

A. Arrange an ultrasound scan of the thyroid


B. Arrange a 99mtechnetium scan of the thyroid
C. Start treatment with levothyroxine
D. Start carbimazole
E. Do not start treatment now but recheck thyroid function tests in 3 months

This patient has primary hypothyroidism, as indicated by the low T4 level with raised TSH. She has clinical
symptoms of hypothyroidism and treatment with levothyroxine is required. The positive anti-thyroid
peroxidase antibody result suggests an autoimmune cause such as Hashimoto’s disease. Imaging of the
thyroid gland is unlikely to provide any additional information, and such cases can generally be managed in
the primary care setting. Carbimazole is used in the treatment of hyperthyroidism.

777447225 87
Prepared by: Osama M. Alsurmi

Question 7
Which of the following statements is true of the drug carbimazole in the
treatment of Graves’ disease?

A. It reduces peripheral conversion of thyroxine (T4) to triiodothyronine (T3)


B. There is a 30% risk of agranulocytosis
C. The risk of relapse is more than 30% within 2 years of stopping the drug
D. It is preferable to propylthiouracil in treating hyperthyroidism in breastfeeding mothers
E. It is associated with an increased risk of thyroid malignancy

The antithyroid drugs carbimazole and propylthiouracil act by inhibiting the iodination of tyrosine and
thereby reducing thyroid hormone synthesis. Since episodic relapse and remission is the most common
pattern in the natural history of Graves’ thyrotoxicosis, carbimazole is usually given for 12–18 months in
the anticipation that the disease will be in remission when the drug is discontinued. However,
thyrotoxicosis recurs in at least 50% of patients within 2 years of stopping treatment. Subtotal
thyroidectomy or radioactive iodine therapy may then be preferred. Agranulocytosis is a rare side-effect of
carbimazole, occurring in 0.2% of patients; they are therefore advised to undergo an urgent full blood count
in the event of a sore throat or fever. Although carbimazole and propylthiouracil are only excreted in small
amounts in breast milk, propylthiouracil is thought to be safer.

777447225 88
Prepared by: Osama M. Alsurmi

Question 8
Serum calcitonin levels are useful in monitoring response to treatment of
which thyroid condition?

A. Medullary carcinoma
B. Thyroid lymphoma
C. Riedel’s thyroiditis
D. Follicular carcinoma
E. Hashimoto’s thyroiditis

Medullary carcinoma arises from the parafollicular C cells of the thyroid. Tumour cells secrete calcitonin,
and serum levels can be used to monitor response to treatment. The condition presents in middle age with a
firm thyroid mass, and treatment is by total thyroidectomy. Thyroid function tests are normal and there is
no role for radio-iodine therapy.

777447225 89
Prepared by: Osama M. Alsurmi

Question 9
Which one of the following is not employed in the investigation and
treatment of papillary carcinoma of the thyroid?

A. Fine needle aspiration of the thyroid


B. Carbimazole
C. Total thyroidectomy
D. Radioactive iodine
E. Measurement of serum thyroglobulin

‘Differentiated’ (papillary and follicular) thyroid carcinomas usually present as palpable nodules for which
the initial investigation of choice is fine needle aspiration cytology. They are usually treated by total
thyroidectomy followed by a large dose of 131iodine to ablate any residual thyroid tissue. Subsequent long-
term treatment with levothyroxine at a dose sufficient to suppress thyroid-stimulating hormone secretion is
required, and recurrence can be detected by secretion of thyroglobulin into the blood. Papillary and
follicular carcinoma have 20-year survival rates of 95% and 60% respectively.

777447225 90
Prepared by: Osama M. Alsurmi

Question 10
All of the following except one are causes of hypogonadotrophic
hypogonadism. Which is the exception?

A. Pituitary adenoma
B. Anorexia nervosa
C. Cystic fibrosis
D. Autoimmune gonadal failure
E. Excessive physical exercise

In hypogonadotrophic hypogonadism there is deficiency of follicle-stimulating hormone and luteinising


hormone, the gonadotrophins. This may be due to structural disease of the pituitary or hypothalamus (e.g.
pituitary adenoma), or to ‘functional’ gonadotrophin deficiency secondary to other influences (e.g. chronic
illness, stress, malnutrition, excessive exercise). Primary gonadal failure, on the other hand, is associated
with elevated gonadotrophin levels due to a loss of negative feedback (hypergonadotrophic hypogonadism).

777447225 91
Prepared by: Osama M. Alsurmi

Question 11
Which of the following statements is true regarding post-menopausal
oestrogen-containing hormone replacement therapy (HRT)?

A. Treatment is usually continued until the age of 65


B. Women with an intact uterus should receive both oestrogen and progesterone
C. It has been demonstrated to reduce the risk of ischaemic heart disease
D. It is a useful adjuvant treatment for thrombophilia
E. It reduces the risk of breast cancer

Oestrogen replacement in post-menopausal women reduces vasomotor symptoms (e.g. hot flushes, sweats)
and prevents osteoporotic fractures. There is, however, an increased risk of breast and endometrial cancer,
thromboembolic disease, ischaemic heart disease and stroke. For these reasons, HRT should not be
continued long-term, and many authorities recommend that treatment is only continued beyond the age of
50 if there are unacceptable symptoms of oestrogen deficiency. In women with an intact uterus, unopposed
oestrogen therapy is more likely to cause endometrial malignancy; combined cyclical treatment with
progesterone is therefore recommended to induce regular withdrawal bleeds.

777447225 92
Prepared by: Osama M. Alsurmi

Question 12
Which of the following is least likely to cause hyperprolactinaemia?

A. Bromocriptine
B. Metoclopramide
C. Pituitary adenoma
D. Pregnancy
E. Hypothyroidism

High serum prolactin levels may be physiological (e.g. stress, pregnancy) or pathological. Prolactin
secretion from the anterior pituitary is normally inhibited by dopamine from the hypothalamus. Pituitary
tumours can cause hyperprolactinaemia by directly secreting prolactin (prolactinoma) or by compressing
the infundibular stalk and thereby interfering with the inhibitory effect of dopamine. Anti-dopaminergic
drugs (e.g. antiemetics and antipsychotics) can also cause high prolactin levels. Bromocriptine, on the other
hand, is a dopamine agonist used in the treatment of prolactinoma. Primary hypothyroidism leads to
increased hypothalamic thyrotrophin-releasing hormone secretion by negative feedback, which in turn
stimulates prolactin release from the pituitary.

777447225 93
Prepared by: Osama M. Alsurmi

Question 13
All of the following measures except one have a role in the treatment of
polycystic ovarian syndrome (PCOS). Which is the exception?

A. Metformin
B. Combined oral contraceptive pill
C. Gliclazide
D. Weight loss
E. Cyproterone acetate

PCOS is associated with insulin resistance and obesity. Weight loss and treatment with insulin-sensitising
drugs, such as metformin, are beneficial; gliclazide is a sulphonylurea that enhances insulin secretion but
not insulin sensitivity, and has no role in PCOS. The combined oral contraceptive pill enables cyclical
delivery of oestrogen and progesterone, which regulates menstrual cycles and improves hirsutism by
suppressing androgen production from the ovary. Cyproterone acetate is an androgen receptor antagonist
(which also has progesterone agonist effects) and is useful in combating hirsutism in PCOS.

777447225 94
Prepared by: Osama M. Alsurmi

Question 14
Which one of the following statements is true of Klinefelter’s syndrome?

A. It is associated with a 47XYY karyotype


B. Affected individuals suffer from hypogonadotrophic hypogonadism
C. There is an increased incidence of gynaecomastia
D. Treatment with growth hormone is usually required
E. Testes are usually absent

Klinefelter’s syndrome affects 1 in 1000 males and is usually associated with the 47XXY karyotype. There
is dysgenesis and subsequent fibrosis of the seminiferous tubules, and affected individuals usually have
small, firm testes. Tall stature (due to androgen deficiency and failure to close the epiphyseal growth plates)
is common, and growth hormone treatment is not required. Impaired testicular function results in androgen
deficiency, which in turn leads to elevated follicle-stimulating hormone and luteinising hormone levels due
to loss of negative feedback (hypergonadotrophic hypogonadism). The lack of androgens causes
gynaecomastia, failure of development of secondary sex characteristics and infertility.

777447225 95
Prepared by: Osama M. Alsurmi

Question 15
Which one of the following statements regarding parathyroid hormone
(PTH) is false?

A. It is a polypeptide hormone
B. It inhibits 1α-hydroxylation of vitamin D in the kidney
C. It stimulates osteoclastic bone resorption
D. It promotes calcium resorption by the renal tubules
E. Its release is inhibited by hypercalcaemia

PTH is a polypeptide hormone secreted by the parathyroid glands in response to low serum calcium levels.
Its actions lead to an increase in serum calcium. This is mediated by increased calcium resorption from
bone and increased renal absorption of calcium. Vitamin D derived from sunlight and from the diet initially
undergoes 25-hydroxylation in the liver to 25-hydroxy-vitamin D, which then undergoes 1α-hydroxylation
in the kidney to form active 1,25-dihydroxy-vitamin D. PTH stimulates the latter step. Activated vitamin D
increases gut absorption of calcium, thereby raising calcium levels further.

777447225 96
Prepared by: Osama M. Alsurmi

Question 16
All of the following except one are recognised causes of hypercalcaemia.
Which is the exception?

A. Primary hyperparathyroidism
B. Secondary hyperparathyroidism
C. Tertiary hyperparathyroidism
D. Multiple myeloma
E. Squamous cell bronchial carcinoma

Primary hyperparathyroidism is usually due to a parathyroid adenoma, but can be caused by parathyroid
hyperplasia or, in rare cases, parathyroid carcinoma. Elevated parathyroid hormone (PTH) levels result in
hypercalcaemia. Secondary hyperparathyroidism occurs as a response to hypocalcaemia (e.g. in chronic
renal failure), and calcium levels are therefore normal or low. Prolonged secondary hyperparathyroidism
can ultimately lead to autonomous PTH hypersecretion with consequent hypercalcaemia (tertiary
hyperparathyroidism). Malignancy can cause elevated serum calcium due to bony metastases,
paraneoplastic phenomena (mediated by PTH-related peptide, e.g. in squamous cell bronchial carcinoma)
or myeloma.

777447225 97
Prepared by: Osama M. Alsurmi

Question 17
Which cluster of features is classically seen in multiple endocrine neoplasia
(MEN) type 2 (Sipple’s syndrome)?

A. Hyperparathyroidism, pituitary adenoma, pancreatic neuro-endocrine tumour


B. Hyperparathyroidism, follicular thyroid carcinoma, phaeochromocytoma
C. Hyperparathyroidism, medullary thyroid carcinoma, phaeochromocytoma
D. Hyperparathyroidism, small cell lung carcinoma, colon carcinoma
E. Phaeochromocytoma, renal cell carcinoma, cerebellar haemangioblastoma

MEN syndromes are rare autosomal dominant disorders characterised by tumours or hyperplasia in multiple
glands. Differentiated thyroid cancers (e.g. follicular carcinoma) are not a feature. Option A describes MEN
type 1, and option E describes von Hippel–Lindau syndrome.

777447225 98
Prepared by: Osama M. Alsurmi

Question 18
A 41-year-old woman with Graves’ disease undergoes subtotal
thyroidectomy. Twenty-four hours post-operatively she complains that she
is unable to straighten her fingers. She also describes tingling around her
mouth. What pattern of abnormalities is most likely to be seen on blood
testing?

A. High calcium, high phosphate, low parathyroid hormone (PTH)


B. High calcium, low phosphate, high PTH
C. Low calcium, high phosphate, low PTH
D. Low calcium, high phosphate, high PTH
E. Low calcium, high phosphate, normal PTH
This patient has hypoparathyroidism resulting from damage to the parathyroid glands during her
thyroidectomy. Since the net effect of PTH is to increase serum calcium and decrease phosphate, this
results in hypocalcaemia and hyperphosphataemia. Carpopedal spasm is a manifestation of tetany and is
characteristic of hypocalcaemia. In carpal spasm, the metacarpophalangeal joints are flexed and the
interphalangeal joints extended. Other features of low calcium include tingling in the hands and feet and
around the mouth, stridor and seizures. Chvostek’s and Trousseau’s signs may be positive.

777447225 99
Prepared by: Osama M. Alsurmi

Question 19
A 43-year-old woman attends her general practitioner with a 1-year history
of progressive weight gain and malaise. She bruises easily and has trouble
climbing stairs. She also has increased facial hair. On examination she has a
body mass index of 32 kg/m2. Her abdomen is obese with visible striae,
although her arms and thighs appear wasted. What is the most likely
diagnosis?

A. Conn’s syndrome
B. Cushing’s syndrome
C. Addison’s disease
D. Phaeochromocytoma
E. Congenital adrenal hyperplasia
Cushing’s syndrome is caused by excessive glucocorticoid levels, which may be exogenous or endogenous
in origin. Addison’s disease results from primary adrenocortical insufficiency. In Conn’s syndrome there is
excess mineralocorticoid secretion by an adrenal adenoma. Phaeochromocytoma arises from a tumour of
catecholamine-producing enterochromaffin cells, most commonly from the adrenal medulla.

777447225 100
Prepared by: Osama M. Alsurmi

Question 20
What is the most useful initial diagnostic investigation in a patient with
suspected Cushing’s syndrome?

A. Random plasma cortisol measurement


B. Short Synacthen test
C. Overnight dexamethasone suppression test
D. Oral glucose tolerance test
E. Twenty-four-hour urine metanephrines
The first step is to establish whether or not the patient has Cushing’s syndrome; further investigations are
then required to determine the cause. Random plasma cortisol levels are unhelpful in view of the diurnal
variation in cortisol secretion. The overnight dexamethasone suppression test involves administration of 1
mg of the synthetic glucocorticoid dexamethasone at bedtime with measurement of plasma cortisol as soon
as practicable after the patient wakes up the next morning. In healthy individuals, endogenous
adrenocorticotrophic hormone (ACTH) secretion is inhibited and plasma cortisol is therefore suppressed to
< 60 nmol/L. Failure to suppress endogenous cortisol secretion is consistent with Cushing’s syndrome.
Alternative screening tests include measurement of cortisol in a 24-hour urine sample, late-night salivary
cortisol, or the cortisol:creatinine ratio in an overnight urine sample.

777447225 101
Prepared by: Osama M. Alsurmi

Question 21
Which of the following clinical and serum biochemical parameters is most
consistent with Addison’s disease?

A. High blood pressure, low sodium, high potassium


B. Low blood pressure, low sodium, high potassium
C. Low blood pressure, high sodium, high potassium
D. Low blood pressure, high sodium, low potassium
E. High blood pressure, high sodium, high potassium

In Addison’s disease there is primary adrenocortical insufficiency. Mineralocorticoid (aldosterone)


deficiency leads to hypotension, hyponatraemia and hyperkalaemia. Glucocorticoid (cortisol) deficiency
causes hypotension and hyponatraemia. Patients can present with chronic features (e.g. fatigue, weight loss,
anorexia) or with an acute adrenal crisis associated with circulatory collapse. The latter is a medical
emergency requiring urgent treatment with intravenous fluids and hydrocortisone.

777447225 102
Prepared by: Osama M. Alsurmi

Question 22
Which of the following drugs is the most appropriate medical treatment for
hypertension associated with Conn’s syndrome?

A. Bendroflumethiazide
B. Atenolol
C. Perindopril
D. Amlodipine
E. Spironolactone

In Conn’s syndrome there is an aldosterone-secreting adrenal adenoma. Aldosterone promotes sodium and
water retention and increases renal potassium excretion. High mineralocorticoid levels therefore result in
hypertension and hypokalaemia. The diuretic spironolactone is a mineralocorticoid receptor antagonist and
is therefore first-line treatment. Surgical resection of the diseased adrenal gland is usually recommended.

777447225 103
Prepared by: Osama M. Alsurmi

Question 23
A 17-year-old woman with a strong family history of diabetes mellitus is
brought to the hospital accident and emergency department after collapsing
at home. On arrival she has a Glasgow Coma Scale score of 9/15. Laboratory
blood glucose is 1.2 mmol/L. Plasma insulin levels are elevated and C-
peptide levels are low. Her condition improves following treatment with
intravenous dextrose. She has had three similar episodes over the preceding
6 months. What is the most likely diagnosis?

A. Insulinoma
B. Alcohol excess
C. Diabetes mellitus
D. Administration of exogenous insulin
E. Sulphonylurea ingestion
Endogenous insulin is co-secreted with C-peptide (both are formed from the cleavage of pro-insulin from
the pancreas). High insulin levels associated with low C-peptide levels during hypoglycaemia therefore
indicate an exogenous origin of the insulin. The family history of diabetes in this patient suggests that she
may have access to a relative’s insulin at home. In insulinoma and sulphonylurea ingestion, the
hyperinsulinaemia is endogenous and C-peptide levels are therefore also elevated. Hypoglycaemia in
alcohol excess and hypopituitarism is not due to increased insulin secretion; insulin and C-peptide levels
are therefore low.

777447225 104
Prepared by: Osama M. Alsurmi

Question 24
A 29-year-old man is referred to the medical outpatient clinic with a 3-
month history of polyuria. He describes increasing thirst and drinks large
volumes of fluid. On examination he appears euvolaemic. A water
deprivation test reveals the following results:
The most likely diagnosis is:

A. Diabetes mellitus
B. Nephrogenic diabetes insipidus
C. Cranial diabetes insipidus
D. Addison’s disease
E. Syndrome of inappropriate antidiuretic hormone secretion (SIADH)
Diabetes insipidus is characterised by the production of large volumes of dilute urine. It is secondary to
deficient production/release of ADH by the hypothalamus/posterior pituitary (cranial diabetes insipidus) or
to reduced responsiveness of the renal tubules to ADH (nephrogenic diabetes insipidus). In healthy
individuals, water deprivation leads to increased ADH secretion. This causes increased water retention by
the kidneys with the resultant formation of concentrated urine (> 600 mmol/kg) and maintenance of normal
plasma osmolality (280–296 mmol/kg). In diabetes insipidus, the kidneys are unable to retain water despite
water deprivation. Plasma therefore becomes concentrated (> 300 mmol/kg) and the urine is not maximally
concentrated (i.e. it is < 600 mmol/kg). In cranial diabetes insipidus (ADH deficiency) the urine does
concentrate after administration of the ADH analogue DDAVP, but in nephrogenic diabetes insipidus
(ADH resistance) there is a minimal response to DDAVP.

777447225 105
Prepared by: Osama M. Alsurmi

Question 25
A 75-year-old woman attends the hospital accident and emergency
department with palpitations. Her past medical history is unremarkable,
although she has recently developed a troublesome tremor. ECG reveals
atrial fibrillation (AF) with a ventricular rate of 145 bpm. Blood pressure is
129/68 mmHg. Blood tests show biochemical evidence of thyrotoxicosis.
What would be the most appropriate initial treatment?

A. Digoxin
B. Propranolol
C. Verapamil
D. Amiodarone
E. DC cardioversion
Thyrotoxicosis is an important cause of atrial fibrillation. Of the agents listed above, the non-selective β-
blocker propranolol is best for AF associated with hyperthyroidism and is also useful for many other
symptoms of hyperthyroidism (particularly tremor) that are mediated by increased sympathetic activity.
Ventricular rate in thyrotoxic patients with AF characteristically responds poorly to digoxin. Prolonged use
of amiodarone can lead to hyper- or hypothyroidism. Acute DC cardioversion is reserved for those patients
who are haemodynamically compromised by the rapid ventricular rate. Two other key elements of this
patient’s management will be definitive treatment of her thyrotoxicosis and anticoagulation
(thromboembolic complications are particularly common in thyrotoxic AF).

777447225 106
Prepared by: Osama M. Alsurmi

Question 26
What is the most common cause of Cushing’s syndrome?

A. Pituitary adenoma
B. Adrenal adenoma
C. Adrenal carcinoma
D. Chronic glucocorticoid therapy
E. Ectopic adrenocorticotrophic (ACTH) syndromes

Prolonged administration of glucocorticoids in supraphysiological quantities (e.g. in patients with asthma or


inflammatory bowel disease) is the most common cause of Cushing’s syndrome. In patients who have not
received glucocorticoid therapy, pituitary adenoma is the most common cause (Cushing’s disease).

777447225 107
Prepared by: Osama M. Alsurmi

Diabetes Mellitus

777447225 108
Prepared by: Osama M. Alsurmi

Question 1
Which one of the following is a physiological action of insulin?

A. Increased lipolysis
B. Increased gluconeogenesis
C. Increased ketogenesis
D. Decreased glycogenolysis
E. Decreased protein synthesis

Insulin is an anabolic hormone, and its secretion increases in response to a rise in blood glucose. Its actions
are to reduce blood glucose levels and promote energy storage (therefore only D is true), inhibit
ketogenesis, and increase fat and protein synthesis.

777447225 109
Prepared by: Osama M. Alsurmi

Question 2
All of the following statements except one are true of type 2 diabetes.
Which is the exception?

A. It is frequently detected incidentally on blood testing


B. It is associated with the metabolic syndrome
C. Ketoacidosis is a common complication
D. There is an increased risk of ischaemic heart disease
E. The prevalence is higher than that of type 1 diabetes

Diabetic ketoacidosis develops in type 1 diabetes because of severe insulin deficiency but is rarely a feature
of type 2 diabetes, as sufficient insulin secretion to suppress lipolysis is usually retained. Type 2 diabetes
has a much higher prevalence and is a major risk factor for cardiovascular disease. Unlike type 1 diabetes,
it is often asymptomatic and detected incidentally. One of the characteristics of type 2 diabetes is insulin
resistance. The combination of type 2 diabetes, hypertension, hyperlipidaemia and central obesity
constitutes the metabolic syndrome, also known as the insulin resistance syndrome, which strongly
predisposes to ischaemic heart disease.

777447225 110
Prepared by: Osama M. Alsurmi

Question 3
A 58-year-old lady presents to her general practitioner with a 6-month
history of general malaise and fatigue. Routine blood tests are carried out
and she is found to have a fasting glucose of 7.5 mmol/L. A repeat fasting
glucose 1 month later is 7.4 mmol/L. The most appropriate next step would
be to:

A. Repeat a fasting glucose measurement in 1 month’s time


B. Perform an oral glucose tolerance test
C. Measure C-peptide levels
D. Explain to the patient that she has diabetes mellitus
E. Reassure the patient that she has normal glucose tolerance

Criteria for diagnosing diabetes mellitus are shown in Box 21.10. C-peptide is the connecting peptide which
links the A and B chains of insulin, and is secreted from the islet ß-cells with insulin in equimolar
concentrations. It can be measured in the assessment of hypoglycaemia to establish whether excess insulin
originates from an exogenous or endogenous source and, occasionally, to determine endogenous insulin
secretory reserve in a patient with diabetes; however, it is not used to make the diagnosis of diabetes.

777447225 111
Prepared by: Osama M. Alsurmi

Question 4
What is the most informative method of assessing a patient’s glycaemic
control in the diabetic outpatient clinic?

A. Performing urine dipstick analysis for glucose


B. Performing urine dipstick analysis for protein
C. Measuring plasma glucose
D. Measuring glycated haemoglobin (HbA1C)
E. Careful analysis of the patient’s dietary intake according to carbohydrate exchanges

Glycated haemoglobin (HbA1C) is formed by the non-enzymatic attachment of glucose to haemoglobin.


Measurement of HbA1C provides an estimate of glycaemic control over the lifespan of red blood cells.
(Red cells have a lifespan of 120 days, but the turnover of the pool in the circulation means that
HbA1C reflects blood glucose over the previous 8 weeks.) The target level is usually 7% or less (i.e. less
than or equal to approximately 50 mmol/mol in the newer IFCC (International Federation of Clinical
Chemistry and Laboratory Medicine) units), as this reduces the risk of developing microvascular
complications. Pre-prandial blood glucose measurements are useful to patients at home for monitoring
glycaemic control and adjusting insulin doses. Assessment of urine glucose is of limited value since it
varies according to the renal threshold.

777447225 112
Prepared by: Osama M. Alsurmi

Question 5
A 23-year-old man with type 1 diabetes presents to the hospital accident
and emergency department with a 24-hour history of nausea, poor appetite
and feeling unwell. Initial investigations show the following results:
What is the most immediate management priority?

A. Administration of intravenous insulin and intravenous saline


B. Administration of intravenous insulin and intravenous potassium
C. Administration of intravenous insulin and broad-spectrum antibiotics
D. Administration of intravenous insulin and intravenous sodium bicarbonate
E. Central line insertion to enable accurate monitoring of fluid balance

In diabetic ketoacidosis (DKA), severe insulin deficiency leads to hyperglycaemia and the generation of
ketone bodies. The hyperglycaemia results in glycosuria and an osmotic diuresis, through which patients
can become severely dehydrated. The initial priorities in managing DKA are therefore administration of
intravenous saline and insulin. Although serum potassium levels are often initially high, patients are usually
depleted in total body potassium, and intravenous infusion of potassium should be given once potassium
falls below 5.0 mmol/L. Sodium bicarbonate is rarely needed, may be dangerous and should be
administered only under expert supervision. Central venous access may be required to assist with fluid
management but is not the most urgent step. Antibiotic therapy would be required if there was evidence of
overt infection as the precipitating cause of the DKA.

777447225 113
Prepared by: Osama M. Alsurmi

Question 6
A 63-year-old man with diabetes presents to the hospital accident and
emergency department with a 4-day history of feeling tired and unwell. He
complains of severe thirst. On examination he appears dehydrated with a
pulse of 116 bpm and blood pressure of 114/68 mmHg. After routine
investigations, he is found to be in a hyperglycaemic hyperosmolar state.
Which one of the following statements regarding this condition is true?

A. Higher doses of insulin are usually required compared to diabetic ketoacidosis


B. Subcutaneous heparin should be administered within the first 24 hours
C. There is usually a profound metabolic acidosis
D. People with types 1 and 2 diabetes are affected equally
E. Large volumes of intravenous sodium bicarbonate are often needed

Hyperglycaemic hyperosmolar state (HHS; previously known as hyperosmolar non-ketotic hyperglycaemia


(HONK)) principally affects patients with type 2 diabetes. It is characterised by severe hyperglycaemia and
marked dehydration, but patients are not acidotic because ketone production is not increased. Smaller doses
of insulin are required than for treatment of diabetic ketoacidosis. There is a high risk of thromboembolic
complications in HSS due to the severe dehydration, and low molecular weight heparin should therefore be
given.

777447225 114
Prepared by: Osama M. Alsurmi

Question 7
Features of hypoglycaemia include all of the following except one. Which is
the exception?

A. Sweating
B. Kussmaul breathing
C. Confusion
D. Coma
E. Tachycardia

Sweating and tachycardia are stimulated by sympathetic activation in hypoglycaemia. Confusion and
altered consciousness result from neuroglycopenia. Kussmaul breathing (air hunger) is a feature of diabetic
ketoacidosis as a response to the metabolic acidosis.

777447225 115
Prepared by: Osama M. Alsurmi

Question 8
A 73-year-old lady with type 2 diabetes presents to hospital with an acute
myocardial infarction (MI). She normally takes metformin. What is the most
appropriate way to maintain glycaemic control in the peri-infarct period?

A. Continue metformin at the current dose


B. Continue metformin at a higher dose
C. Stop metformin and monitor blood glucose readings over the first 24 hours
D. Stop metformin and commence a sulphonylurea
E. Stop metformin and commence intravenous insulin

Studies suggest that strict glycaemic control in the peri-infarct period improves long-term prognosis after
an MI. This is usually achieved with insulin, often in the form of an intravenous infusion (sliding scale).
The research most often cited is the DIGAMI study (diabetes mellitus,insulin glucose infusion
in acute myocardial infarction), which showed a reduction in mortality of 25% in patients treated with an
insulin infusion, compared with standard treatment over a follow-up period of 3.5 years (though this has not
been confirmed in subsequent studies). Oral anti-diabetic agents are usually discontinued until the recovery
period after MI. Metformin in particular should be withheld because of the small risk of lactic acidosis in
the event of impaired systemic perfusion.

777447225 116
Prepared by: Osama M. Alsurmi

Question 9
What is the correct perioperative management of an insulin-treated type 2
diabetic patient undergoing a laparotomy?

A. Position the patient at the head of the operation list and omit insulin until after the operation
B. Administer his or her usual subcutaneous insulin pre-operatively and monitor blood glucose
C. Administer half of his or her usual subcutaneous dose pre-operatively and monitor blood glucose
D. Start an intravenous infusion of dextrose and insulin in place of the patient’s usual therapy
E. Give him or her one dose of an oral anti-diabetic agent to prevent high blood glucose levels
perioperatively

Adequate control of hyperglycaemia is important in the perioperative period. For type 2 diabetic patients
undergoing minor operations, it is reasonable to place them first on the operating list and to omit their
morning oral anti-diabetic agent or insulin until after the operation. However, for type 2 diabetic patients
undergoing major surgery, and for all type 1 diabetic patients, blood glucose should be controlled by means
of an intravenous insulin infusion.

777447225 117
Prepared by: Osama M. Alsurmi

Question 10
A patient with type 2 diabetes and a body mass index of 34 kg/m2 has
suboptimal glycaemic control with dietary therapy alone. Which of the
following would be the most appropriate first-line pharmacological
treatment?

A. Gliclazide
B. Metformin
C. Sitagliptin
D. Insulin
E. Exenatide
Sulphonylureas (e.g. gliclazide) act by stimulating endogenous insulin secretion. A major side-effect of this
class of drug is weight gain, and so they would not generally be considered first-line treatment in obese
patients. Metformin (a biguanide) works by increasing insulin sensitivity and does not cause weight gain, so
is a better choice here. Gliptins (e.g. sitagliptin, vildagliptin) are oral agents that inhibit the enzyme
dipeptidyl peptidase 4 (DPP-4). DPP-4 is responsible for the breakdown of glucagon-like peptide 1 (GLP-
1), an incretin hormone that stimulates insulin secretion in response to glucose. The gliptins therefore
increase insulin secretion in a glucose-dependent manner. They are weight-neutral and have few side-
effects but are not usually prescribed as first-line treatment. Exenatide is a synthetic form of GLP-1 that
potentiates glucose-induced insulin secretion and also induces weight loss in most patients. Insulin
treatment is sometimes required in type 2 diabetes, but tends to be initiated when glycaemic control with
oral agents is suboptimal.

777447225 118
Prepared by: Osama M. Alsurmi

Question 11
Which of the following statements about the thiazolidinediones (glitazones)
is false?

A. They act by binding to peroxisome proliferator-activated receptor-γ (PPAR-γ)


B. They do not increase insulin secretion
C. They are particularly useful in diabetic patients with left ventricular dysfunction
D. They are useful agents in obese type 2 diabetic patients
E. They rarely cause hypoglycaemia

The thiazolidinedione class of drugs, also known as glitazones or TZDs, includes rosiglitazone and
pioglitazone. They act by binding to a nuclear receptor called peroxisome proliferator-activated receptor-γ
(PPAR-γ), particularly in fat cells. This results in altered expression of genes involved in metabolism and
thereby enhances insulin sensitivity. Insulin secretion itself is not affected. They are useful in obese patients
with type 2 diabetes, in whom insulin resistance is often severe. Although they do cause weight gain, this is
usually in subcutaneous rather than intra-abdominal adipose tissue and is therefore thought to be
metabolically ‘benign’. TZDs can cause salt and water retention, and are therefore contraindicated in heart
failure. Pioglitazone reduces myocardial infarctions and strokes, so may be beneficial in patients with
cardiovascular disease. Rosiglitazone may, however, slightly increase the risk of MI.

777447225 119
Prepared by: Osama M. Alsurmi

Question 12
Which one of the following drugs is correctly paired with its predominant
side-effect?

A. A
B. B
C. C
D. D
E. E

Sulphonylureas (e.g. gliclazide, glipizide) stimulate insulin secretion and hence cause weight gain and can
induce hypoglycaemia. Metformin increases insulin sensitivity and does not cause hypoglycaemia, but
frequently causes nausea and diarrhoea. It is also contraindicated in patients with renal impairment, liver
dysfunction, symptomatic heart failure and sepsis because of the risk of lactic acidosis.

777447225 120
Prepared by: Osama M. Alsurmi

Question 13
What is the biggest cause of mortality in diabetic patients?

A. Ischaemic heart disease


B. Hypoglycaemia
C. Renal failure
D. Diabetic ketoacidosis
E. Infection

Macrovascular disease is the major cause of mortality in diabetes, particularly myocardial infarction and
stroke. Macrovascular disease also causes significant morbidity in the form of coronary heart disease,
cerebrovascular disease and peripheral vascular disease. Microvascular complications include diabetic
retinopathy, nephropathy and neuropathy. Their incidence is related to poor glycaemic control and causes
serious morbidity but is not the major cause of death.

777447225 121
Prepared by: Osama M. Alsurmi

Question 14
Which of the following fundal abnormalities is not typical of diabetic
retinopathy?

A. Hard exudates
B. Microaneurysms
C. Flame haemorrhages
D. Cotton wool spots
E. Venous loops and beading

Flame haemorrhages are a feature of hypertensive retinopathy but are not usually a feature of diabetic
retinopathy. Hard exudates (due to leakage from abnormal capillaries), blot and dot haemorrhages, and
cotton wool spots (caused by areas of retinal ischaemia) are found in both diabetic and hypertensive
retinopathy. Microaneurysms and venous beading occur in diabetic retinopathy.

777447225 122
Prepared by: Osama M. Alsurmi

Question 15
In the absence of contraindications, which of the drugs in the following list
is the best antihypertensive agent in a diabetic patient with
microalbuminuria?

A. Bendroflumethiazide
B. Atenolol
C. Ramipril
D. Doxazosin
E. Amlodipine
Diabetic nephropathy is a major cause of end-stage renal failure. Its development is related to poor
glycaemic control. The presence of microalbuminuria heralds the development of overt nephropathy. The
risk of progression can be reduced by good glycaemic control and aggressive treatment of elevated blood
pressure. Angiotensin-converting enzyme (ACE) inhibitors (e.g. ramipril) and angiotensin II receptor
blockers (e.g. irbesartan) provide greater benefit from equal blood pressure lowering than other
antihypertensive agents. They reduce intraglomerular pressure and urinary protein excretion and are
therefore the preferred agents in patients with microalbuminuria.

777447225 123
Prepared by: Osama M. Alsurmi

Question 16
All of the following statements except one are true of diabetic neuropathy.
Which is the exception?

A. The incidence can be reduced by strict glycaemic control


B. Motor, sensory and autonomic nerves can all be involved
C. The majority of patients are asymptomatic
D. Charcot joints are a recognised complication
E. It is typically associated with painful foot ulcers

Like all diabetic microvascular complications, the development of neuropathy can be prevented or delayed
by good glycaemic control. Sensory, motor and autonomic nerves can all be affected, but patients are often
asymptomatic, particularly in the early stages. Sensory neuropathy classically gives rises to a ‘glove and
stocking’ distribution of sensory loss distally. Motor involvement can cause progressive proximal muscle
weakness, commonly accompanied by severe pain. Motor and sensory function of individual cranial and
peripheral nerves can also be affected (mononeuropathy and mononeuritis multiplex). Peripheral sensory
neuropathy can ultimately result in grossly deforming neuroarthropathy of the feet (Charcot joint) due to
loss of pain sensation. Although neuropathy can give rise to foot ulcers, these are typically painless (unlike
ischaemic ulcers).

777447225 124
Prepared by: Osama M. Alsurmi

Question 17
All of the following statements about type 1 diabetes are true except one.
Which is the exception?

A. Viral infection has been implicated in the aetiology


B. There is an association with pernicious anaemia
C. There is destruction of the pancreatic islet ß-cells
D. Treatment with insulin is essential
E. Concordance rates between monozygotic twins are greater than 90%

Type 1 diabetes is mediated by autoimmune destruction of the insulin-producing ß-cells in the pancreas.
Treatment with insulin is therefore essential to prevent uncontrolled hyperglycaemia and ketoacidosis. The
condition is thought to be triggered by environmental factors (e.g. viral infection) in genetically susceptible
individuals. However, concordance between monozygotic twins is only 35% in type 1 diabetes, compared
with rates approaching 100% in type 2 diabetes. Type 1 diabetes is associated with other autoimmune
conditions, including pernicious anaemia, Addison’s disease and thyroid disease.

777447225 125
Prepared by: Osama M. Alsurmi

Question 18
You are asked to review a 67-year-old lady on the ward who has insulin-
treated type 2 diabetes and is complaining of nausea, sweating and
palpitations. Blood glucose strip testing gives a reading of 2.6 mmol/L.
What would be the most appropriate initial management?

A. Intravenous dextrose
B. Intramuscular glucagon
C. Intravenous glucagon
D. Oral carbohydrate
E. Taking a blood sample for a formal laboratory measurement of blood glucose
Although the accuracy of blood glucose strip testing kits is variable, this patient has clinical features and a
glucose reading consistent with hypoglycaemia, so prompt treatment is required. If a patient is conscious
and able to swallow, then oral carbohydrate is the treatment of choice. If not, then intravenous dextrose or
intramuscular glucagon should be administered. Laboratory samples should be obtained (for glucose,
insulin, C-peptide, toxicology drug screen) before treatment in patients with unexplained suspected
‘spontaneous’ hypoglycaemia, as this avoids potentially complex attempts to reproduce the hypoglycaemia
later. However, hypoglycaemia in diabetic patients who are treated with insulin or with a sulphonylurea can
almost always be assumed to be related to their treatment.

777447225 126
Prepared by: Osama M. Alsurmi

Alimentary tract & pancreatic disease

777447225 127
Prepared by: Osama M. Alsurmi

Question 1
Which one of the following features on clinical examination would suggest a
kidney rather than a spleen as the cause of a palpable left upper quadrant
mass?

A. Cannot palpate above it


B. Presence of a notch
C. Dullness to percussion
D. Extends towards the right iliac fossa
E. Rounded mass

The other findings are all characteristic of an enlarged spleen.

777447225 128
Prepared by: Osama M. Alsurmi

Question 2
The major enzyme responsible for protein digestion in the small bowel is:

A. Trypsin
B. Amylase
C. Lipase
D. Cholecystokinin
E. Pepsin

Amylase is involved in carbohydrate digestion. Lipase cleaves long-chain triglycerides, yielding fatty
acids and monoglycerides. Cholecystokinin (CCK) is a hormone with important effects on gut
motility. Pepsin is active in the stomach, not small bowel, where it plays a modest role in protein
digestion.

777447225 129
Prepared by: Osama M. Alsurmi

Question 3
The enzyme amylase is principally responsible for the breakdown of which
one of the following?

A. Protein
B. Cholesterol
C. Glycogen
D. Bile salts
E. Triglycerides

777447225 130
Prepared by: Osama M. Alsurmi

Question 4
A 55-year-old lady with a 3-month history of progressive dysphagia for
solids and weight loss is investigated by upper gastrointestinal endoscopy:
What is the most likely diagnosis?

A. Achalasia
B. Oesophageal candidiasis
C. External oesophageal compression by bronchial carcinoma
D. Oesophageal carcinoma
E. Myasthenia gravis

The endoscopy shows ulceration and erosion, as well as an exophytic growth typical of an oesophageal
carcinoma. None of the other options would produce this appearance. Oesophageal carcinoma must
always be looked for first in patients presenting with dysphagia for solids, unless there are other features
clearly indicating a neurological or motility problem, e.g. muscle wasting, speech disturbance or ptosis.

777447225 131
Prepared by: Osama M. Alsurmi

Question 5
In which of the following causes of dysphagia would oesophageal
manometry be most likely to provide the diagnosis?

A. Myasthenia gravis
B. Bulbar palsy
C. Pseudobulbar palsy
D. Carcinoma of oesophagus
E. Achalasia

Upper gastrointestinal endoscopy is, in general, the investigation of choice in dysphagia to exclude
structural causes and allow biopsy with or without dilatation of any suspicious strictures. However,
oesophageal manometry is useful in achalasia where the characteristic pattern of high-pressure, non-
relaxing lower oesophageal sphincter with poorly contractile body of oesophagus confirms the diagnosis.

777447225 132
Prepared by: Osama M. Alsurmi

Question 6
Which of the following drugs is least likely to be implicated in a patient
presenting with vomiting?

A. Erythromycin
B. Codeine phosphate
C. Atenolol
D. Cisplatin
E. Digoxin

Many antibiotics, analgesics and cytotoxins may lead to vomiting. Side-effects of atenolol include
tiredness, impotence and nightmares, but not vomiting.

777447225 133
Prepared by: Osama M. Alsurmi

Question 7
In patients presenting with acute upper gastrointestinal haemorrhage, all
of the following factors except one are associated with an increased risk of
death. Which is the exception?

A. Pulse > 100 bpm


B. Mallory–Weiss tear as the cause of bleeding
C. Visible but non-bleeding vessel present at endoscopy
D. Age over 60 years
E. Associated renal failure

Mortality risk in patients presenting with acute upper gastrointestinal bleed is defined by the Blatchford
score, which takes into account age, features of shock, comorbidity, diagnosis and endoscopic findings.
Mallory–Weiss tear is the diagnosis associated with the most favourable outcome.

777447225 134
Prepared by: Osama M. Alsurmi

Question 8
A 71-year-old lady presents with a 4-week history of weight loss and
progressively worsening jaundice with pruritus. On examination she is
deeply icteric and has a palpable gallbladder. What is the most likely
diagnosis?

A. Chronic cholecystitis
B. Hepatocellular carcinoma
C. Ascending cholangitis
D. Carcinoma of pancreas
E. Amoebic abscess

This is a fairly typical presentation of pancreatic carcinoma (apart from the rare sign of palpable
gallbladder). The diagnosis will be confirmed with abdominal ultrasound or CT. According to
‘Courvoisier’s law’, a palpable gallbladder in a jaundiced patient is unlikely to be due to gallstones
because chronic inflammation leads to a shrunken gallbladder. It is usually the result of extrahepatic
biliary obstruction by a carcinoma of the head of pancreas.

777447225 135
Prepared by: Osama M. Alsurmi

Question 9
Which of the following conditions/drugs is least likely to cause oral
ulceration?

A. Coeliac disease
B. Nicorandil
C. Behçet’s disease
D. Herpes simplex virus
E. Psoriasis

777447225 136
Prepared by: Osama M. Alsurmi

Question 10
Which one of the following factors does not predispose to gastro-
oesophageal reflux disease (GORD)?

A. Pregnancy
B. High intake of coffee
C. Weight loss
D. Delayed gastric emptying
E. Hiatus hernia

Weight gain may precipitate or aggravate symptoms by increasing intra-abdominal pressure, whereas
weight loss may improve symptoms.

777447225 137
Prepared by: Osama M. Alsurmi

Question 11
Which one of the following statements is true of Barrett’s oesophagus?

A. It refers to neoplastic change in the normal squamous cells of the lower oesophagus
B. The majority of patients with the condition ultimately develop carcinoma of the oesophagus
C. It is associated with increased alcohol intake
D. Patients with the condition should undergo regular endoscopic surveillance with biopsies
E. Antireflux surgery has been shown to induce regression of the condition

Barrett’s oesophagus refers to metaplastic rather than neoplastic change. Although it is a major risk
factor for oesophageal adenocarcinoma, the vast majority of patients do not go on to develop this
condition. Neither potent acid suppression nor antireflux surgery has been shown to induce regression or
prevent progression of Barrett’s oesophagus. Surveillance is expensive and cost-effectiveness studies
have been conflicting, but currently it is recommended that patients with columnar-lined oesophagus
(CLO) without dysplasia should undergo endoscopy every 3–5 years and those with low-grade dysplasia
at 6-monthly intervals.

777447225 138
Prepared by: Osama M. Alsurmi

Question 12
Which one of the following statements is true concerning carcinoma of the
oesophagus?

A. The overall 5-year survival is approximately 40%


B. CT is superior to endoscopic ultrasound in local staging
C. A history of painful, intermittent dysphagia for solids and liquids is the usual presenting
complaint
D. The presence of hoarseness suggests mediastinal invasion
E. Symptomatic relief of swallowing difficulties is best achieved by palliative radiotherapy

The overall 5-year survival rate is very poor at 14%; even after ‘potentially curative’ surgery, it remains
only 30%. Endoscopic ultrasound is the most sensitive modality for local staging, as CT tends to
understage tumours. Dysphagia is classically persistent and progressive, and is initially worse for solids.
Pain is often absent due to destruction of mucosal innervation by the tumour. Endoscopic therapy by
laser or argon plasma coagulation (APC) or stent insertion is the major method of improving swallowing.
Mediastinal invasion with involvement of the recurrent laryngeal nerve may cause hoarseness.

777447225 139
Prepared by: Osama M. Alsurmi

Question 13
A 45-year-old male presents with a 10-month history of recurrent,
‘gnawing’ epigastric pain. The pain does not radiate and can be absent for
up to a month at a time but, when present, seems to be relieved by eating.
He has also experienced occasional vomiting. With which of the following
diagnoses would his symptoms fit best?

A. Chronic peptic ulcer


B. Biliary colic
C. Chronic pancreatitis
D. Mesenteric ischaemia
E. Crohn’s disease
Of the possibilities above, chronic peptic ulcer fits best with the pattern of symptoms. The three
characteristics of ulcer pain are the recurrent, episodic nature, a relationship to food and localisation to
the epigastrium. Occasional vomiting occurs in about 40%. It is worth noting, though, that the history is
actually a poor predictor of the presence of an ulcer. The pain of chronic pancreatitis and mesenteric
ischaemia tends to be precipitated rather than relieved by eating, as does the pain of biliary colic,
particularly with fatty foods. It would be unusual for Crohn’s disease to produce pain well localised to
the epigastrium.

777447225 140
Prepared by: Osama M. Alsurmi

Question 14
Which one of the following statements concerning the
bacterium Helicobacter pyloriis true?

A. It is a Gram-positive rod
B. It produces an enzyme called ‘urease’ that lowers the surrounding pH
C. It exclusively colonises gastric-type epithelium
D. Strains expressing the enzyme cagA are less often associated with disease
E. On colonising the stomach, it induces increased secretion of somatostatin from D cells

H. pylori is a Gram-negative spiral-shaped rod. The enzyme ‘urease’ produces ammonia from urea,
thereby raising the surrounding pH and making the environment less hostile for the bacterium. It is true
that H. pylori colonises only gastric-type epithelium and is, therefore, only found in the duodenum in
association with patches of gastric metaplasia. CagA is one of several well-recognised virulence factors
(see Fig. 22.33 in the link below). Colonisation is associated with depletion of somatostatin (from D cells)
but stimulates release of gastrin (from G cells), thus leading to increased acid secretion.

777447225 141
Prepared by: Osama M. Alsurmi

Question 15
Which of the following is least likely to occur as a complication of partial
gastrectomy?

A. Weight gain
B. Flushing, sweating and lightheadedness after meals with large carbohydrate content
C. Postprandial diarrhoea and abdominal discomfort
D. Anaemia
E. Early satiety

Weight loss rather than weight gain is common. The usual cause is reduced dietary intake due to early
satiety from a small gastric remnant. Both diarrhoea and anaemia may be multifactorial. Rapid gastric
emptying of food of high osmotic potential may produce large fluid shifts into the small intestine,
provoking autonomic symptoms. This is known as ‘dumping syndrome’.

777447225 142
Prepared by: Osama M. Alsurmi

Question 16
A 68-year-old lady presents to her GP with tiredness and weight loss and is
found to have iron deficiency anaemia. On examination there is a palpable
mass in the right iliac fossa. Which of the following is the most appropriate
next investigation to establish the diagnosis?

A. Flexible sigmoidoscopy
B. Abdominal CT scan
C. Abdominal ultrasound
D. Colonoscopy
E. Serum carcinoembryonic antigen (CEA)
The combination of occult gastrointestinal bleeding, weight loss and a palpable right iliac fossa mass in a
patient of this age group should lead to strong suspicion of colonic (caecal) carcinoma and should be
investigated as a matter of urgency by colonoscopy. Colonoscopy is preferable to barium enema in most
instances, as it has better sensitivity and specificity, and allows biopsy of lesions and polypectomy. If the
diagnosis is confirmed, then a CT scan may be useful for detecting hepatic metastases. CEA is neither
sensitive nor specific for diagnosing colorectal cancer, but in those patients with a raised CEA who go on
to have resection of the tumour, it may help in follow-up to detect early recurrences.

777447225 143
Prepared by: Osama M. Alsurmi

Question 17
All of the following except one are recognised risk factors for gastric
carcinoma. Which is the exception?

A. Autoimmune chronic gastritis


B. Zollinger–Ellison syndrome
C. Cigarette smoking
D. H. pylori infection
E. Familial adenomatous polyposis

A reduction in the acidity of the stomach (hypochlorhydria or achlorhydria) appears to increase gastric
cancer risk, perhaps by permitting colonisation of the stomach by nitrite-reducing bacteria, which form
carcinogenic N-nitroso-compounds from nitrates. Autoimmune chronic gastritis and H. pylori infection
may both lead to achlorhydria, unlike Zollinger–Ellison syndrome, which markedly increases gastric
acidity. Although H. pylori infection normally results in normal or increased acid secretion, 5–10% of
infected individuals in Western countries become hypochlorhydric or achlorhydric.

777447225 144
Prepared by: Osama M. Alsurmi

Question 18
Which one of the following conditions is not associated with coeliac
disease?

A. Type 1 diabetes mellitus


B. Pernicious anaemia
C. Conn’s syndrome
D. Primary biliary cirrhosis
E. Dermatitis herpetiformis

Many other autoimmune conditions are more common in patients with coeliac disease. Conn’s syndrome
is not an autoimmune disorder. It is characterised by hyperkalaemia and high blood pressure, and is
usually caused by overproduction of aldosterone from a benign adenoma of the adrenal glands (‘primary
hyperaldosteronism’).

777447225 145
Prepared by: Osama M. Alsurmi

Question 19
A 43-year-old male presents with a 6-hour history of severe, constant upper
abdominal pain radiating to the back. He drinks approximately 80 units of
alcohol per week. On examination there is marked epigastric tenderness
but no guarding or rebound tenderness. Which of the following tests is
likely to be most useful in reaching a positive diagnosis?

A. Plain abdominal X-ray


B. C-reactive protein
C. Erect chest X-ray
D. ECG
E. Serum amylase
The above account is classical of acute pancreatitis. Pain radiates to the back in 65% of cases; guarding
and rebound tenderness are typically absent in the early stages, as the inflammation is retroperitoneal. A
markedly elevated serum amylase (or lipase) confirms the diagnosis, although in difficult cases,
ultrasound or CT can help by demonstrating evidence of pancreatic oedema and swelling. Abdominal
and erect chest X-rays may exclude other conditions such as intestinal obstruction or perforated viscus,
and an ECG should be performed to exclude atypical presentation of myocardial infarction. C-reactive
protein is not helpful diagnostically but is a useful indicator of severity and prognosis.

777447225 146
Prepared by: Osama M. Alsurmi

Question 20
All of the following except one are uncommon but recognised causes of
acute pancreatitis. Which is the exception?

A. Azathioprine therapy
B. Mumps
C. Hypocalcaemia
D. Hypothermia
E. Seat belt injury

Hypercalcaemia, not hypocalcaemia, is a rare cause of acute pancreatitis. By far the two most common
aetiologies are alcohol and gallstones. Blunt trauma to the abdomen can result in injury to the pancreatic
body as it crosses in front of the vertebral column.

777447225 147
Prepared by: Osama M. Alsurmi

Question 21
Which one of the following is not an adverse prognostic factor in acute
pancreatitis?

A. A
B. B
C. C
D. D
E. E

Although an elevated serum amylase is useful in diagnosing acute pancreatitis, its level has no prognostic
value. The other parameters listed are included in the Glasgow criteria of adverse prognostic factors.

777447225 148
Prepared by: Osama M. Alsurmi

Question 22
Which one of the following would be considered an unexpected clinical
feature in chronic pancreatitis?

A. Steatorrhoea
B. Weight loss
C. Pain relieved by drinking alcohol
D. Recurrent hypoglycaemia
E. Reticulate skin pigmentation over the back

Hyperglycaemia due to the development of diabetes mellitus, rather than hypoglycaemia, is a common
feature of chronic pancreatitis. Reticulate pigmentation over the abdomen and back may develop due to
chronic application of a hot water bottle to relieve pain, producing erythema ab igne (‘fireside tartan’).

777447225 149
Prepared by: Osama M. Alsurmi

Question 23
Which one of the following physical consequences of malabsorption
is incorrectlymatched with the corresponding vitamin or mineral deficiency?

A. A
B. B
C. C
D. D
E. E

Vitamin C deficiency produces bleeding gums, poor wound healing and perifollicular or petechial
bruising (scurvy). Peripheral neuropathy may be due to vitamin B12 deficiency.

777447225 150
Prepared by: Osama M. Alsurmi

Question 24
Which one of the following statements regarding carcinoma of the pancreas
is true?

A. The majority of tumours are amenable to potentially curative resection


B. The body of the pancreas is the most common site of origin
C. Coeliac plexus neurolysis is useful in relieving pruritus
D. The liver is a common site for metastatic deposits
E. The prognosis is better than for carcinoma of the ampulla of Vater

Pancreatic carcinoma has a dismal prognosis; only 15–20% of tumours are amenable to curative
resection and, even in patients who undergo complete resection, 5-year survival is only 20%. Around
two-thirds of tumours arise from the head of the pancreas; these frequently involve the common bile
duct, resulting in obstructive jaundice. Coeliac plexus neurolysis may be useful in relieving pain, which
often proves extremely difficult to control with other measures. Ampullary carcinomas behave less
aggressively than pancreatic carcinomas and have a more favourable outlook.

777447225 151
Prepared by: Osama M. Alsurmi

Question 25
Which one of the following pathological features would favour a diagnosis
of ulcerative colitis over Crohn’s disease?

A. Presence of fistulae
B. Proximal colitis with rectal sparing
C. Lymphoid aggregates and microgranulomas
D. Ulceration within the small intestine
E. Inflammation limited to the mucosa
There are several important pathological differences between ulcerative colitis (UC) and Crohn’s disease
(CD), including but not confined to the following:
 UC only involves the colon, while CD can involve any part of the gastrointestinal tract from mouth
to anus.
 In UC, inflammation is confluent and more severe distally (thus it invariably involves the rectum
but may spread proximally to involve the remainder of the colon). In CD, changes are patchy with
areas of inflammation interrupted by islands of normal mucosa. (This may produce the
characteristic ‘skip lesion’.)
 In UC, the inflammatory process is superficial, confined to the mucosa and sparing deeper layers of
the bowel wall, while in CD, inflammation is deep and seen through all layers of the bowel wall.
(Deep ulcers may penetrate through the bowel wall to initiate fistulae.)

777447225 152
Prepared by: Osama M. Alsurmi

Question 26
Which of the following would be least expected to provoke a relapse (‘flare’)
in ulcerative colitis?

A. Loss of job
B. Cigarette smoking
C. Ankle sprain treated with oral diclofenac
D. Viral laryngitis
E. Course of antibiotics for falsely suspected urinary tract infection

Ulcerative colitis is more common in non-smokers and, in fact, cessation of smoking can provoke a
relapse of symptoms. Emotional stress, intercurrent illness, non-steroidal anti-inflammatory drugs
(NSAIDs) and antibiotics are all well-recognised factors that may precipitate flares.

777447225 153
Prepared by: Osama M. Alsurmi

Question 27
A 19-year-old male presents with a 4-week history of bloody diarrhoea and
lower abdominal pain associated with low-grade fever, malaise and
anorexia. Which of the following investigations would be least helpful in
establishing a diagnosis of ulcerative colitis?

A. Stool culture and microscopy


B. Sigmoidoscopy and rectal biopsy
C. Blood cultures and serological tests of infection
D. Serum albumin
E. Examination of stool for Clostridium difficile toxin
Other causes of bloody diarrhoea, abdominal pain and fever must be excluded. The investigations listed
above will allow the exclusion of these other causes, and the sigmoidoscopy and rectal biopsy should
confirm the diagnosis. Serum albumin is a marker of disease severity in ulcerative colitis but does not
contribute to the diagnosis. It is suppressed in any condition producing an acute phase response.
Hypoalbuminaemia is the least useful diagnostically, since it would be reduced as a non-specific finding
in both inflammation and infection. This avoids having to assume a diagnosis.

777447225 154
Prepared by: Osama M. Alsurmi

Question 28
The patient from the previous question has several clinical and laboratory
parameters indicating severe ulcerative colitis. Which one of the following
is true regarding the management of this attack?

A. Nutritional support, if required, should be provided intravenously rather than enterally


B. The patient should be treated with intravenous corticosteroids
C. Surgery should not be considered until he has had at least 10 days of maximal medical therapy
D. Prophylaxis of venous thromboembolism with heparin should be avoided
E. Sulfasalazine overcomes the need for surgery in around 80% of patients who fail to respond to
corticosteroids

Intravenous corticosteroids are given as a constant infusion in the treatment of severe ulcerative colitis.

777447225 155
Prepared by: Osama M. Alsurmi

Question 29
On the fourth day of medical treatment, the patient from the previous
question does not show any signs of improvement. A plain abdominal X-ray
is performed:
Which one of the following statements is true?

A. It is unusual for this complication to occur in the first attack of colitis


B. The original diagnosis of ulcerative colitis was probably incorrect
C. The patient should be referred for urgent colectomy
D. No further action should be taken but the X-ray should be repeated in 24 hours
E. Azathioprine treatment may avoid the need for surgery in patients with this complication

The X-ray shows a grossly dilated colon due to severe ulcerative colitis. Toxic megacolon occurs most
commonly during the first attack of colitis and is associated with a high risk of perforation. It is,
therefore, an indication for urgent colectomy.

777447225 156
Prepared by: Osama M. Alsurmi

Question 30
Which one of the following statements concerning intestinal complications
of inflammatory bowel disease is incorrect?

A. In ulcerative colitis, colonic perforation may occur without the development of toxic megacolon
B. In Crohn’s disease, enterovesical fistulation may present with pneumaturia
C. The risk of developing colon cancer in ulcerative colitis is related to both the extent and the
duration of colitis
D. Both conditions may present with acute life-threatening haemorrhage
E. Diarrhoea in ulcerative colitis is often due to enteroenteric fistulae

Fistulous connections between loops of affected bowel (enteroenteric), or between bowel and bladder
(enterovesical) or vagina (enterovaginal), are specific complications of Crohn’s disease and do not occur
in ulcerative colitis. Patients with extensive active colitis of more than 8 years’ duration are at increased
risk of colon cancer and should, therefore, have regular colonoscopic surveillance.

777447225 157
Prepared by: Osama M. Alsurmi

Question 31
Which one of the following systemic complications of inflammatory
bowel disease isnot related to the activity of bowel disease?

A. Conjunctivitis
B. Primary sclerosing cholangitis
C. Erythema nodosum
D. Arthralgia
E. Pyoderma gangrenosum

Primary sclerosing cholangitis and cholangiocarcinoma are both more common in ulcerative colitis but
appear to be unrelated to intestinal disease activity. The other complications listed tend to be more of a
problem during active bowel disease.

777447225 158
Prepared by: Osama M. Alsurmi

Question 32
Concerning the management of Crohn’s disease, which one of the following
statements is false?

A. Due to high rates of relapse, infliximab therapy should be combined with disease-modifying
agents
B. Maintenance of remission is improved by cessation of cigarette smoking
C. Corticosteroids are preferred to methotrexate for maintaining remission
D. Surgical intervention should be conservative to minimise loss of viable intestine
E. Azathioprine is an effective maintenance treatment
Chronic corticosteroid treatment is avoided due to the high incidence of complications with long-term
use and poor efficacy in preventing relapse. All of the other statements are true and are useful learning
points. Azathioprine and methotrexate are both used in maintenance of remission. Infliximab is highly
effective in inducing remission in active Crohn’s but relapse commonly occurs around 12 weeks after
treatment. It should therefore be given in combination with either azathioprine or methotrexate. Surgery
is not curative in Crohn’s disease and repeated surgery may lead to the development of short bowel
syndrome. Cessation of cigarette smoking is more important than any drug treatment in maintaining
remission.

777447225 159
Prepared by: Osama M. Alsurmi

Question 33
A 32-year-old woman presents with severe recurrent epigastric pain.
Endoscopy reveals multiple ulcers in the duodenum, stomach and
oesophagus. There is no history of ingestion of non-steroidal anti-
inflammatory drugs (NSAIDs) and tests forH. pylori are negative. Serum
gastrin levels are grossly elevated. What is the likely diagnosis?

A. Covert NSAID abuse


B. Carcinoid syndrome
C. Zollinger–Ellison syndrome
D. Crohn’s disease
E. Biliary reflux
In Zollinger–Ellison syndrome there is severe peptic ulceration from gastric acid hypersecretion due to a
gastrin-producing non-ß-cell islet tumour of the pancreas (‘gastrinoma’). Ulcers are often multiple and
refractory to standard therapy, and may occur in unusual places such as oesophagus or duodenum. It
comprises part of the multiple endocrine neoplasia (MEN) type 1 syndrome. Inactivation of pancreatic
enzymes and bile salts by increased acidity in the small intestine can result in diarrhoea (seen in more
than one-third of patients) and steatorrhoea. High-dose proton pump inhibitor therapy heals ulcers,
alleviates diarrhoea and has largely removed the need for total gastrectomy.

777447225 160
Prepared by: Osama M. Alsurmi

Question 34
Which one of the following clinical features is least suggestive of
irritable bowel syndrome (IBS)?

A. Colicky abdominal pain relieved by defecation


B. Sensation of incomplete defecation
C. Symptoms disturbing sleep
D. Alternating diarrhoea and constipation
E. Passage of mucus

Nocturnal symptoms are uncommon in IBS and pain that wakens a patient from sleep is often a useful
pointer to organic disease. Other features that would raise doubt as to the diagnosis include rectal
bleeding and weight loss.

777447225 161
Prepared by: Osama M. Alsurmi

Question 35
Regarding the management of irritable bowel syndrome (IBS), which one of
the following statements is true?

A. Patients with constipation as the predominant symptom benefit most from tricyclic
antidepressant therapy
B. Elimination diets are the mainstay of treatment
C. Probiotics are of no benefit
D. Reassurance alone may lead to resolution of symptoms
E. All patients should be advised to increase dietary fibre

Anxiety over the possibility of serious organic disease, particularly cancer, is often a major contributing
factor to symptoms in IBS and many patients respond to simple explanation and reassurance. Such
patients have a far better long-term outlook than those who fail to respond.

777447225 162
Prepared by: Osama M. Alsurmi

Question 36
A 75-year-old lady presents with a 2-hour history of sudden-onset severe
abdominal pain and per rectum bleeding. Other than a stroke 2 years ago
she keeps well. She has been on aspirin and ramipril since her stroke and
takes digoxin for an irregular heart rhythm. On examination she is very
distressed; her abdomen is soft with no evidence of peritonism. Bowel
sounds are audible. An arterial blood gas shows a metabolic acidosis. What
is the likeliest diagnosis?

A. Ruptured aortic aneurysm


B. Acute small bowel ischaemia
C. Perforated peptic ulcer
D. Diverticulitis
E. Irritable bowel syndrome
The rapidity of onset, history of vascular disease and atrial fibrillation (not on warfarin), paucity of
abdominal findings despite severe symptoms, and metabolic acidosis all point to acute small bowel
ischaemia. Given her history of atrial fibrillation, a likely aetiology would be a thromboembolism from
the heart.

777447225 163
Prepared by: Osama M. Alsurmi

Question 37
With regard to colonic polyps, which one of the following statements is
true?

A. They usually present with lower gastrointestinal bleeding or anaemia


B. The size of the polyp has no bearing on the risk of malignant change
C. Asymptomatic polyps found at colonoscopy should be biopsied but not removed
D. Polyps with a villous architecture carry a higher risk of malignant change than those with
tubular architecture
E. The polyps in Peutz–Jeghers syndrome are predominantly colonic adenomas
Nearly all forms of colorectal carcinoma develop from adenomatous polyps, usually over a period of 5–
10 years, although certain features such as increased size and villous architecture increase the risk for
malignant change. Adenomas are usually asymptomatic and are often found incidentally during
colonoscopy; when discovered, they should be removed whenever possible to reduce the risk of
developing colorectal cancer. Peutz–Jeghers syndrome is characterised by the presence of non-neoplastic
hamartomatous polyps in the colon and small bowel (although it is associated with increased risk of
certain malignancies, e.g. breast, ovary, colon).

777447225 164
Prepared by: Osama M. Alsurmi

Question 38
A patient recently commenced on a new medication complains of
constipation. Which of the following drugs is most likely to be implicated?

A. Erythromycin
B. Digoxin
C. Orlistat
D. Metformin
E. Verapamil

Constipation is a side-effect of various calcium channel blockers but particularly verapamil. Diarrhoea is
a common side-effect of macrolide antibiotics, especially erythromycin. Likewise, metformin is far more
likely to cause diarrhoea than constipation. Side-effects of digoxin include nausea, vomiting, arrhythmia
and disturbed vision, but not constipation. Orlistat is a lipase inhibitor that reduces absorption of dietary
fat and is used in the management of obesity. Side-effects include liquid or oily stools and faecal urgency
rather than constipation. Digoxin can cause diarrhoea in certain patients.

777447225 165
Prepared by: Osama M. Alsurmi

Question 39
The urea breath test is useful in the diagnosis of which one of the following
conditions?

A. Bacterial overgrowth
B. Helicobacter pylori infection
C. Lactose intolerance
D. Fat malabsorption
E. Coeliac disease

Urea breath testing is, in most cases, the best test for detecting H. pyloriinfection, as it is accurate, simple
and non-invasive. Bacterial overgrowth and lactose intolerance may be detected by the glucose hydrogen
breath test and lactose hydrogen breath test, respectively. Fat malabsorption can be diagnosed by either
3-day faecal fat assessment or the 14C-triolein breath test, although these are not widely available. Tests
for coeliac disease include serology (tissue transglutaminase, tTG) and duodenal biopsy.

777447225 166
Prepared by: Osama M. Alsurmi

Question 40
A 48-year-old male presents with a history of having passed a large amount
of fresh blood in his stool. He reports feeling dizzy and lightheaded. Heart
rate on admission is measured at 120 beats per minute and the blood
pressure is recording repeatedly at 80/50 mmHg. You assess that the
patient is likely to have suffered an acute lower gastrointestinal
haemorrhage and is manifesting features of shock. Which of the following
conditions would be least likely to account for this presentation?

A. Diverticular disease
B. Angiodysplasia
C. Meckel’s diverticulum
D. Bowel ischaemia
E. Anal fissure
Anal fissure does not cause severe acute lower gastrointestinal bleeding but rather small amounts of fresh
rectal bleeding associated with pain during defecation. The volume of blood lost in order to make the
patient hypovolaemic would be unlikely to be due to an anal fissure. Of the other causes, diverticular
disease is common among older patients. However, in a male patient of this age, the possibility that the
bleeding is arising from a colorectal cancer must be considered.

777447225 167
Prepared by: Osama M. Alsurmi

Question 41
Which of the following statements is true concerning coeliac disease in
adults?

A. Patients are intolerant of maize and rice


B. Peak onset is in the third decade
C. Almost all patients who do not follow a gluten-free diet are symptomatic
D. Adhering to a strict gluten-free diet increases the risk of metabolic bone disease
E. There is an increased risk of T-cell lymphoma

Patients are intolerant of wheat, rye, barley and, to a lesser extent, oats, but not rice, maize or potatoes.
Peak onset is in infancy and early childhood; in adults the highest incidence occurs in the 40–59 age
group. Around 50% of patients are asymptomatic and many remain undiagnosed. Metabolic bone disease
is less common in patients who stick strictly to a gluten-free diet. There is an increased risk of T-cell
lymphoma, as well as small bowel carcinoma and squamous carcinoma of the oesophagus, although the
absolute risk of developing these malignancies is very low.

777447225 168
Prepared by: Osama M. Alsurmi

Question 42
A 57-year-old woman presents with a 12-month history of slowly
progressive dysphagia for solids, eased by drinking liquids. She also
experiences intermittent episodes of severe chest pain. A barium swallow is
performed:
What is the most likely diagnosis?

A. Diffuse oesophageal spasm


B. Achalasia
C. Systemic sclerosis
D. Carcinoma of oesophagus
E. Myasthenia gravis
The barium swallow shows the characteristic appearance of a dilated barium-filled oesophagus with fluid
level, distal tapering and a closed lower oesophageal sphincter. Clues in the history include the long
duration of symptoms, the easing of swallowing difficulties by drinking liquids, and the intermittent
episodes of severe chest pain that are due to oesophageal spasm. Diffuse oesophageal spasm produces
transient rather than slowly progressive dysphagia. Myasthenia gravis, as with other neuromuscular
causes of dysphagia, tends to be worse for liquids rather than solids. In systemic sclerosis there is usually
a history of severe heartburn. An endoscopy may still be necessary to exclude oesophageal carcinoma.

777447225 169
Prepared by: Osama M. Alsurmi

Liver& biliary tract disease

777447225 170
Prepared by: Osama M. Alsurmi

Question 1
Which of the following is not a function of the liver?

A. Synthesis of clotting factors


B. Production of vitamin K
C. Production of glucose during fasting
D. Storage of copper
E. Catabolism and/or excretion of potentially toxic compounds

Vitamins are, by definition, not synthesised in the human body so adequate dietary intake and
gastrointestinal absorption of vitamin K are essential. Vitamin K is a fat-soluble vitamin and absorption is
impaired in obstructive jaundice, resulting in a prolonged prothrombin time that is corrected by vitamin K
administration.

777447225 171
Prepared by: Osama M. Alsurmi

Question 2
A hospital inpatient is noted to have an elevated gamma-glutamyl
transferase (GGT) with otherwise normal liver function tests. She is taking
the following regular medications. Which one is most likely to have caused
this abnormality?

A. Omeprazole
B. Paracetamol
C. Phenytoin
D. Ibuprofen
E. Sodium valproate

Phenytoin is a microsomal enzyme-inducing drug; all drugs of this type may cause an isolated increase in
the microsomal enzyme GGT. This is a reflection of hepatic enzyme induction and does not necessitate any
change to therapy. A list of enzyme-inducing drugs can be found in the link below. It is important to learn
them, as they have interactions with other drugs, such as reducing the effectiveness of the oral
contraceptive pill and increasing the toxicity of paracetamol in overdose.

777447225 172
Prepared by: Osama M. Alsurmi

Question 3
Of the following conditions, which is the most likely cause of jaundice
accompanied by a seven-fold elevation of alanine aminotransferase and
two-fold elevation of alkaline phosphatase?

A. Acute viral hepatitis


B. Acute cholecystitis
C. Gilbert’s syndrome
D. Haemolysis
E. Pancreatic carcinoma

This is the classic picture of hepatocellular jaundice.

777447225 173
Prepared by: Osama M. Alsurmi

Question 4
Which one of the following coagulation factors depends on vitamin K for
complete synthesis in the liver?

A. Factor IV
B. Factor V
C. Factor VIII
D. Factor IX
E. Factor XI

The vitamin K-dependent clotting factors are II, VII, IX and X. No-one has ever devised a decent
mnemonic for this fact, so you just have to remember it. Here they are again: 2, 7, 9 and 10.

777447225 174
Prepared by: Osama M. Alsurmi

Question 5
A patient presents to her GP after noticing that the whites of her eyes have
become yellow. Direct questioning also reveals that her urine has been very
dark for several days. Which of the following diagnoses is the most likely
explanation for these symptoms?

A. Carotenaemia
B. Cholestatic jaundice
C. Hyperbilirubinaemia secondary to Gilbert’s syndrome
D. Rhabdomyolysis
E. Dehydration
Cholestatic jaundice is the only option here that explains both yellow sclerae and dark urine. Conjugated
bilirubin from the liver enters the blood because of a failure of bile flow. As conjugated bilirubin (unlike
unconjugated bilirubin) is water-soluble, it is filtered by the kidneys and appears in the urine. More
information on cholestatic jaundice, including causes and clinical features, is provided in the link below.
Carotenaemia, caused by eating too much carotene, is almost always benign and causes yellowing of the
skin but not the sclera. Gilbert’s syndrome describes a congenital impairment of glucuronyl transferase, that
leads to reduced hepatic uptake of bilirubin from the blood. This therefore causes unconjugated
hyperbilirubinaemia, so the sclerae are yellow but the urine is not dark. In contrast, rhabdomyolysis causes
large amounts of myoglobin to enter the urine, which can make the urine appear black but does not cause
jaundice. Dehydration results in highly concentrated urine but does not cause jaundice.

777447225 175
Prepared by: Osama M. Alsurmi

Question 6
A 47-year-old man with a history of alcoholic cirrhosis is well known on the
general medical ward after recurrent admissions for drainage of ascites. He
presents with confusion, fever (38.5oC), ascites and abdominal pain. Which
of the following investigations is most important to establish the diagnosis?

A. Stool culture and microscopy for ova, cysts and parasites


B. Diagnostic laparoscopy
C. Abdominal ultrasound scan
D. CT scan of abdomen
E. Ascitic fluid white cell count

This is a classic presentation of spontaneous bacterial peritonitis, a common and life-threatening


complication of cirrhosis.

777447225 176
Prepared by: Osama M. Alsurmi

Question 7
Which of the following is the first-line drug of choice to treat chronic ascites
secondary to hepatic cirrhosis?

A. Furosemide
B. Spironolactone
C. Digoxin
D. Bendroflumethiazide
E. Vasopressin

Aldosterone excess, caused by impaired hepatic steroid metabolism, is a key factor in the pathogenesis of
ascites in chronic liver disease (explained in the link below). Spironolactone is an aldosterone antagonist,
which makes it the best choice to prevent sodium retention in cirrhotic patients. Furosemide (a loop
diuretic) is sometimes added to spironolactone to promote sodium excretion further in the urine.
Bendroflumethiazide (a thiazide diuretic) is not often used to treat ascites because better drugs are
available. Vasopressin is a potent vasoconstrictor and powerfully promotes water retention. It is used in the
management of hepatorenal syndrome. Digoxin is not useful in the management of ascites.

777447225 177
Prepared by: Osama M. Alsurmi

Question 8
Which of the following blood results confers the worst prognosis on a
patient with acute liver failure secondary to paracetamol overdose?

A. Prothrombin time (PT) > 100 secs


B. Aspartate aminotransferase (AST) > 6 times normal
C. Alanine aminotransferase (ALT) > 2.5 times normal
D. Platelets < 100 × 109/L
E. Albumin < 35 g/dL

PT is a better predictor of poor outcome than liver function tests, and is used in standard criteria for
determining the need for liver transplant. The following two prognostic factors (see the link below) have
been shown to confer greater than 90% mortality in acute liver failure secondary to paracetamol overdose:
• H+ > 50 nmol/L (pH < 7.3)
• Creatinine > 300 μmol/L with PT > 100 secs and grade 3 or 4 encephalopathy.

777447225 178
Prepared by: Osama M. Alsurmi

Question 9
Which of the following examination findings is not characteristic of a
palpable spleen?

A. Forms a palpable mass that the examiner cannot get above


B. Is dull to percussion
C. May be balloted
D. Moves with respiration
E. Is felt superficially
Distinguishing a grossly enlarged kidney (for example, in polycystic kidney disease) from splenomegaly
can be tricky.
Spleen:
• It is not possible to ‘get above’ it.
• It moves towards the right iliac fossa as it enlarges.
• It has a palpable notch.
• The overlying percussion note is dull.
Kidney:
• It is possible to get above it.
• It moves downwards as it enlarges.
• It is ballotable.
• It is resonant to percussion.

777447225 179
Prepared by: Osama M. Alsurmi

Question 10
Which of the following statements about primary biliary cirrhosis is true?

A. It is more common in men


B. Transplant is contraindicated because of the high risk of disease recurrence
C. Pruritus is a more common presenting feature than jaundice
D. A positive antinuclear antibody is diagnostic
E. Immunosuppression prevents disease progression

Primary biliary cirrhosis is more common in women by a factor of 9. Pruritus is the most common
presenting complaint and may precede jaundice by years. Positive antinuclear antibodies are found in only
15% of cases, whereas antimitochondrial antibodies are of considerable diagnostic value, present in over
95% of patients. Immunosuppressive drugs are not effective, and treatment mainly palliates symptoms until
transplant becomes necessary. Although 30% of patients will exhibit disease recurrence in a transplanted
liver, this takes years to develop and is not a contraindication to transplant.

777447225 180
Prepared by: Osama M. Alsurmi

Question 11
Which of the following drugs is most useful in a patient with a suspected
variceal haemorrhage, where urgent upper gastrointestinal endoscopy is not
available?

A. Noradrenaline (norepinephrine)
B. Activated protein C
C. Dopamine
D. Terlipressin
E. Ciprofloxacin

Terlipressin lowers the portal pressure and is the drug of choice in this scenario. The somatostatin analogue,
octreotide, is also useful. Noradrenaline (norepinephrine) and dopamine predominantly cause peripheral
vasoconstriction. However, in a patient dying from haemorrhage, fluid replacement, rather than
vasoconstriction, is the best method to correct hypotension. Activated protein C is contraindicated in
haemorrhage.

777447225 181
Prepared by: Osama M. Alsurmi

Question 12
Which of the following viruses is not capable of causing infection without
concurrent infection with another hepatitis virus?

A. Hepatitis A
B. Hepatitis B
C. Hepatitis C
D. Hepatitis D
E. Hepatitis E

Hepatitis D is an incomplete virus and cannot cause infection without concurrent infection with hepatitis B.

777447225 182
Prepared by: Osama M. Alsurmi

Question 13
Which one of the following statements about hepatitis A virus (HAV)
infection is true?

A. It is commonly spread by sexual transmission


B. It may be spread by the faecal–oral route
C. A chronic carrier state occurs in a minority of infected patients
D. No vaccine is available
E. Anti-HAV of IgG type is useful diagnostically

Hepatitis A is not usually spread through blood or conventional sexual contact, and, unlike hepatitis B and
C, does not induce a chronic carrier state. A vaccine is available, and immune serum globulin may be useful
in the management of outbreaks and immunocompromised or pregnant patients. Anti-HAV antibodies of
IgM type may be useful diagnostically.

777447225 183
Prepared by: Osama M. Alsurmi

Question 14
Hepatitis B blood tests in a patient reveal the following:
What is the most likely explanation for these results?

A. Acute hepatitis infection


B. Previous hepatitis infection
C. Chronic hepatitis infection with active viral replication
D. Chronic hepatitis infection with low viral replication
E. Previous immunisation to hepatitis B without previous infection

This is difficult but important to understand. Both the natural history of infection and the serological tests
are explained in the link below. A few things are particularly important to commit to memory. A vaccinated
patient will only have antibody to the surface protein (content of vaccine) (i.e. anti-HBs) and will not have
any positive antigens. A patient who is immune (previously exposed to virus and cleared it) will have
antibodies to the whole virus, i.e. core antibodies (anti-HBc IgG) and surface antibodies (anti-HBs). Note
that HBsAg is present in chronic infection; if continued active viral replication is present then HBeAg is
positive, but if viral replication is low then HBeAg is negative and anti-Hbe antibodies are positive. Anti-
HBc antibodies of the IgM type appear in acute infection; anti-HBc IgG antibodies appear later.

777447225 184
Prepared by: Osama M. Alsurmi

Question 15
Which one of the following statements about infection with hepatitis B virus
(HBV) isfalse?

A. The majority of patients make a full recovery from acute infection


B. Chronic infection may lead to hepatocellular carcinoma
C. HBeAg-negative patients respond less well to treatment
D. Antiviral therapy usually results in resistant mutants of HBV
E. Treatment with interferon is ineffective in chronic infection

Interferon treatment is effective in some patients with hepatitis B, although is seldom used in adult patients
as the majority will clear the virus after an acute infection.

777447225 185
Prepared by: Osama M. Alsurmi

Question 16
Which one of the following statements about hepatitis B infection is true?

A. It may be spread in faeces


B. The route of infection does not affect clinical course
C. It is caused by an RNA virus
D. No vaccine is available
E. It may cause cirrhosis

Hepatitis B is commonly transmitted through blood or sexual contact. The most common route of
transmission worldwide is vertical during childbirth; this route of infection carries a much higher risk of
chronic infection (see the link below), which may ultimately lead to cirrhosis and hepatocellular carcinoma.
A vaccine is given to all health-care staff in the UK.

777447225 186
Prepared by: Osama M. Alsurmi

Question 17
Which one of the following statements is true of hepatitis C infection?

A. It is usually sexually transmitted


B. A minority of patients exposed to the virus will become chronically infected
C. Once infected, cure is impossible
D. Liver transplantation is contraindicated due to the risk of reinfection of the transplanted
liver
E. Acute infection is usually asymptomatic

Hepatitis C is only rarely symptomatic in the acute phase, and is often only discovered once cirrhosis has
developed. About 80% of infected patients develop chronic infection, which usually progresses ultimately
to cirrhosis. Treatment with pegylated interferon and ribavirin can eradicate the virus completely in some
patients. Although reinfection almost inevitably occurs after transplant, this treatment may give otherwise
terminally ill patients a high quality of life for many years, and should not be withheld for this reason.

777447225 187
Prepared by: Osama M. Alsurmi

Question 18
Which one of the following statements about alcoholic liver disease (ALD) is
true?

A. Once cirrhosis develops, abstinence does not affect prognosis


B. There is a genetic predisposition
C. Binge drinkers are more likely to develop ALD than those with a consistent intake
D. The presence of fatty liver (steatosis) on biopsy suggests an alternative diagnosis
E. Transplantation is contraindicated

As in many common liver diseases, studies have revealed a genetic contribution to ALD, as well as
environmental factors. After a presenting complication of cirrhosis, most patients who become abstinent
will be alive after 5 years.

777447225 188
Prepared by: Osama M. Alsurmi

Question 19
A 42-year-old hospital inpatient, with previously normal liver function and
weighing 73 kg, develops abnormal liver function tests. She has taken the
following medications at therapeutic doses in the last 28 days. Which one is
most likely to have caused the problem?

A. Co-amoxiclav
B. Paracetamol
C. Temazepam
D. Morphine
E. Furosemide

Co-amoxiclav (amoxicillin plus clavulanic acid) may cause a cholestatic hepatitis up to 42 days after a
course of treatment has finished. Paracetamol in normal doses can cause liver necrosis but this only occurs
in high-risk patients, such as those with pre-existing alcoholic liver disease.

777447225 189
Prepared by: Osama M. Alsurmi

Question 20
A 37-year-old man presents to his general practitioner complaining of
tiredness, polyuria and thirst. On examination, he is found to have 4 cm
hepatomegaly and a greyish pigmentation to his skin. Urinalysis reveals a
large amount of glycosuria. His past medical history includes osteoarthritis,
for which he takes regular ibuprofen. Which of the following provides the
best unifying diagnosis?

A. Wilson’s disease
B. Diabetes insipidus
C. Acute viral hepatitis
D. Infectious mononucleosis
E. Hereditary haemochromatosis
The combination of joint pain (caused by chondrocalcinosis), diabetes mellitus (‘bronzed’ diabetes),
hepatomegaly and hyperpigmentation is characteristic of hereditary haemochromatosis, one of the most
common inherited diseases in northern Europeans. About 90% of patients are male, perhaps due to the
protective effect of iron loss with menstruation.

777447225 190
Prepared by: Osama M. Alsurmi

Question 21
With reference to the patient in the previous question who has hereditary
haemochromatosis, which of the following is the most appropriate initial
treatment?

A. Liver transplant
B. Weekly venesection of 500 mL blood
C. Warfarin
D. Hepatic lobe resection
E. Metformin

The complications of haemochromatosis arise because of massive iron overload. The aim of treatment is to
remove this excess iron safely, and this is done by weekly venesection until iron levels return to normal.

777447225 191
Prepared by: Osama M. Alsurmi

Question 22
A 34-year-old woman is referred to the gastrointestinal clinic after her
general practitioner discovers abnormal liver function tests (LFTs). She
denies alcohol abuse or sexual risk factors. She received a blood transfusion
in the UK in 2002 following a road traffic accident. Her past medical history
includes obesity (body mass index = 42), type 2 diabetes mellitus and
anxiety/depression. What is the most likely cause of her deranged LFTs?

A. Occult drug overdose


B. Alcoholic liver disease
C. Non-alcoholic fatty liver disease
D. Hepatitis B infection
E. Hepatitis A infection
Obesity is by far the most common cause of non-alcoholic fatty liver disease, which also shows an
association with type 2 diabetes mellitus and the metabolic syndrome. It is the most likely cause in this
patient. The introduction of tests to detect and exclude infected blood units has massively reduced the risk
of transmission of hepatitis B and other viruses through blood transfusion. The chances of hepatitis B
transmission from a unit of blood in the UK in 2002 were around 0.24 per 1 000 000 units.

777447225 192
Prepared by: Osama M. Alsurmi

Question 23
A 25-year-old patient presents with an episode of acute hepatitis from
which he recovers spontaneously and is discharged from hospital with a
presumptive diagnosis of occult drug use. A year later he presents with
fatigue, tremor at rest affecting both hands, and occasional wild,
purposeless movements of his upper limbs. The admitting hospital doctor
notes an abnormality on examination of the eyes:
What is the most likely diagnosis?

A. Ecstasy (MDMA) abuse


B. Budd–Chiari syndrome
C. Wilson’s disease
D. Hereditary haemochromatosis
E. Amanita phalloides poisoning
Recurrent acute hepatitis with parkinsonian features and choreoathetoid movements suggest Wilson’s
disease. The Kayser–Fleischer rings (shown here) around the cornea are pathognomonic but easily missed.
Of the other options, hereditary haemochromatosis usually presents later in life with advanced liver disease.
The others may cause acute liver failure but not Kayser–Fleischer rings or extrapyramidal manifestations.
Budd–Chiari syndrome is hepatic venous thrombosis, Amanita phalloides is a profoundly hepatotoxic
mushroom, and ecstasy (MDMA) is a drug of abuse that may cause liver failure.

777447225 193
Prepared by: Osama M. Alsurmi

Question 24
An overweight 41-year-old woman with a body mass index of 32 presents
with a 5-day history of severe right upper quadrant pain. Initially the pain
was intermittent, lasting for 2 hours then subsiding, but for the past 12
hours it has been constant. On examination she is pyrexial (38.5oC) and
there is tenderness and rigidity in the right upper quadrant. Her white cell
count is 18 × 109/L and C-reactive protein 130 mg/L. Liver function tests
(LFTs) and amylase are within the normal range. Which of the following
diagnoses is most likely?

A. Biliary colic
B. Acute pancreatitis
C. Acute cholecystitis
D. Mirizzi’s syndrome
E. Choledocholithiasis
This is a classic presentation of acute cholecystitis. Biliary colic causes pain but not inflammation, and so
the pyrexia and leucocytosis make this diagnosis less likely. Acute pancreatitis is highly unlikely, given the
normal amylase. Mirizzi’s syndrome (a stricture in the common hepatic duct caused by gallstones impacted
in the cystic duct) and choledocholithiasis (stones in the bile duct) would be expected to cause abnormal
LFTs in a patient with disease severe enough to present with systemic symptoms.

777447225 194
Prepared by: Osama M. Alsurmi

Question 25
Which of the following patterns of symptoms is most characteristic of acute
blockage of the cystic duct by a gallstone?

A. Recurrent epigastric pain, radiating to the back, lasting 2 hours then subsiding
spontaneously
B. Constant severe interscapular pain associated with sweating and hypotension
C. Constant severe epigastric pain and recurrent vomiting
D. Intermittent burning epigastric pain relieved by drinking milk
E. Crushing, heavy epigastric pain radiating to the jaw, associated with sweating, nausea and
a terror of imminent death
‘Biliary colic’ usually lasts for around 2 hours and occurs intermittently. It is most commonly felt in the
epigastrium (70%) or right upper quadrant (20%), and often radiates to the interscapular region or the tip of
the right scapula. Of the other options, D is characteristic of dyspepsia and E is suggestive of an acute
myocardial infarction. B and C are relatively non-specific presentations but serious diagnoses such as
dissecting thoracic aneurysm and acute pancreatitis must be considered.

777447225 195
Prepared by: Osama M. Alsurmi

Question 26
Which of the following investigations is most appropriate for the initial
diagnosis of acute symptomatic gallstone disease?

A. CT of the abdomen
B. Abdominal ultrasound scan
C. Chest X-ray
D. Plain abdominal X-ray
E. Kidney–ureter–bladder (KUB) X-ray

Ultrasound is quick and inexpensive, and does not expose the patient to radiation. Stones may be visualised
within the gallbladder and bile duct, and dilatation of the bile duct may be seen, indicating distal
obstruction of bile flow.

777447225 196
Prepared by: Osama M. Alsurmi

Question 27
Which of the following factors is least likely to be a precipitant of hepatic
encephalopathy?

A. Constipation
B. Upper gastrointestinal bleed
C. Hypokalaemia
D. Hypoglycaemia
E. Urinary tract infection

Hypoglycaemia does not precipitate hepatic encephalopathy but is an important differential diagnosis in a
patient with liver disease presenting with confusion or drowsiness.

777447225 197
Prepared by: Osama M. Alsurmi

Question 28
A jaundiced patient has the following liver function test (LFT) results:
Which one of the following is the most appropriate initial investigation to
identify the underlying problem?

A. Endoscopic retrograde cholangiopancreatography (ERCP)


B. Hepatic portal venography
C. Abdominal X-ray
D. Abdominal ultrasound
E. Liver biopsy
The LFTs show a cholestatic picture. The most useful initial investigation is abdominal ultrasound,
particularly to detect biliary obstruction. Should more detailed imaging of the biliary tree prove necessary,
then either ERCP or magnetic resonance cholangiopancreatography (MRCP) may be helpful. MRCP
(followed if necessary by therapeutic ERCP) is the diagnostic test of choice, as it carries a lower risk of
complications than ERCP. A very useful algorithm for the investigation of patients with jaundice is
provided in the link below.

777447225 198
Prepared by: Osama M. Alsurmi

Question 29
Only one of the following options associates the disease with the correct
initial diagnostic test. Which is it?

A. A
B. B
C. C
D. D
E. E

This is difficult to remember but genuinely useful because AMA has high sensitivity for PBC, being
present in over 95% of cases.

777447225 199
Prepared by: Osama M. Alsurmi

Rhumatology & bone disease

777447225 200
Prepared by: Osama M. Alsurmi

Question 1
Which of the following clinical features is least typical of osteoarthritis?

A. Joint pain aggravated by activity and relieved by rest


B. Involvement of large joints
C. Prolonged morning stiffness
D. Presence of Bouchard’s nodes
E. Palpable crepitus over affected joint

Morning stiffness in osteoarthritis tends to be brief, lasting only a few minutes. Thereafter, pain is related to
movement and weight-bearing, and relieved by rest. This pattern contrasts markedly with inflammatory
arthritides such as rheumatoid arthritis, in which morning stiffness is typically prolonged (lasting longer
than an hour) and symptoms are exacerbated by inactivity.

777447225 201
Prepared by: Osama M. Alsurmi

Question 2
All of the following except one are well-recognised extra-articular features
of rheumatoid arthritis. Which is the exception?

A. Episcleritis
B. Pyoderma gangrenosum
C. Pericarditis
D. Alopecia
E. Pulmonary nodules

All of the above are recognised features of rheumatoid arthritis, with the exception of alopecia. Alopecia is
more commonly associated with systemic lupus erythematosus.

777447225 202
Prepared by: Osama M. Alsurmi

Question 3
All of the following clinical features except one affect the hands and are
characteristic of rheumatoid arthritis (RA). Which is the exception?

A. Bouchard’s nodes
B. Swan neck deformity
C. Piano key deformity
D. Subluxed metacarpophalangeal joints
E. Rupture of the extensor tendons

Bouchard’s nodes are posterolateral hard swellings of the proximal interphalangeal joints, characteristically
seen in nodal generalised osteoarthritis along with Heberden’s nodes (similar swellings arising from the
distal interphalangeal joints). They are not generally seen in RA.

777447225 203
Prepared by: Osama M. Alsurmi

Question 4
All of the following except one are documented side-effects of antirheumatic
drugs. Which is the exception?

A. Renal impairment with ciclosporin


B. Pneumonitis with methotrexate
C. Hypertension with leflunomide
D. Keratitis with hydroxychloroquine
E. Gastritis with non-steroidal anti-inflammatory drugs (NSAIDs)

All of the above associations are true with the exception of D. At very high doses,hydroxychloroquine
(though more commonly chloroquine) may cause retinitis (classically a ‘bull’s eye’ maculopathy) but not
keratitis (inflammation of the cornea).

777447225 204
Prepared by: Osama M. Alsurmi

Question 5
All of the following except one are characteristic features of ankylosing
spondylitis. Which is the exception?

A. Raised erythrocyte sedimentation rate and C-reactive protein


B. Strongly positive rheumatoid factor
C. Association with HLA-B27 histocompatibility antigen
D. Extraspinal features including anterior uveitis and aortic incompetence
E. ‘Bamboo’ spine appearance on X-ray

As one of the seronegative spondarthritides, ankylosing spondylitis has noassociation with seropositivity
for rheumatoid factor. Rheumatoid factor is negative or positive only in low titre.

777447225 205
Prepared by: Osama M. Alsurmi

Question 6
Recognised patterns of psoriatic arthritis include all of the following except
one. Which is the exception?

A. Asymmetric oligoarthritis
B. Arthritis mutilans
C. Symmetrical polyarthritis
D. Atlanto-axial instability
E. Distal interphalangeal joint arthritis

Psoriatic arthritis may present with a variety of different patterns of joint disease. Atlanto-axial subluxation,
however, is a potentially serious manifestation of rheumatoid arthritis and occasionally occurs in other
conditions (e.g. classic ankylosing spondylitis, Down’s syndrome).

777447225 206
Prepared by: Osama M. Alsurmi

Question 7
All of the following statements about gout are true except one. Which is the
exception?

A. Bendroflumethiazide may predispose to gout


B. There is a male preponderance of the disease
C. Allopurinol is often used in the treatment of an acute attack
D. High alcohol intake is a risk factor for the condition in susceptible individuals
E. The monosodium urate crystals deposited within affected joints are negatively birefringent
when viewed under polarised light
Allopurinol is not used to treat acute gout. Management of acute gout includes appropriate analgesia (often
a non-steroidal anti-inflammatory drug), oral colchicine, joint aspiration and intra-articular injection with a
long-acting steroid. The most common side-effects of colchicine are nausea and diarrhoea. Definitive
diagnosis of gout requires demonstration of negatively birefringent needle-shaped crystals in a joint aspirate
when viewed under polarised light. Thiazide diuretics such as bendroflumethiazide reduce urinary excretion
of uric acid and may therefore lead to hyperuricaemia and eventual urate crystal formation. Other drugs that
cause hyperuricaemia include aspirin, ciclosporin, pyrazinamide and alcohol.

777447225 207
Prepared by: Osama M. Alsurmi

Question 8
All of the following except one are associated with pseudogout (calcium
pyrophosphate dihydrate (CPPD) crystal deposition). Which is the
exception?

A. Hyperparathyroidism
B. Acromegaly
C. Wilson’s disease
D. Haemochromatosis
E. Advancing age

Sporadic, familial and metabolic-associated forms of pseudogout exist. The common feature is deposition
of calcium pyrophosphate crystals in the joint (most commonly the knee, followed in frequency by the
wrist, shoulder, ankle and elbow), leading to chondrocalcinosis. Acromegaly is associated with a secondary
osteoarthritis, but not with pseudogout.

777447225 208
Prepared by: Osama M. Alsurmi

Question 9
A 46-year-old builder presents to his general practitioner with a 1-week
history of back pain. Which one of the following features in his presentation
would provide the greatest cause for concern?

A. Sudden onset of the pain while lifting a heavy load at work


B. A history of intermittent episodes of low back pain in the past
C. Pain varying with physical activity but generally relieved by rest
D. Difficulty passing urine
E. Tenderness of the paraspinous muscles

The most important lesson from this question is that there are so-called ‘red flag signs’ (as below) that must
not be ignored, as they may be indicative of serious underlying pathology. All of the other symptoms
described are consistent with simple mechanical low back pain, which is likely to improve with time and
simple analgesia.

777447225 209
Prepared by: Osama M. Alsurmi

Question 10
Which one of the following conditions is associated with a seronegative
arthritis?

A. Systemic lupus erythematosus


B. Rheumatoid arthritis
C. Systemic sclerosis
D. Psoriasis
E. Sjögren’s syndrome

The term ‘seropositive’ refers to arthritis that associates with an increased prevalence of rheumatoid factor.
Psoriasis is one of a family of conditions – the seronegative spondarthritides – that manifest with arthritis
that has no such increased association with rheumatoid factor. Rheumatoid factor has low specificity for
rheumatoid arthritis and occurs with increased prevalence in a wide range of conditions.

777447225 210
Prepared by: Osama M. Alsurmi

Question 11
Which one of the following is a characteristic feature of reactive arthritis
(previously known as Reiter’s disease)?

A. Conjunctivitis
B. Subcutaneous nodules
C. Strong female preponderance
D. Precipitation by streptococcal throat infection
E. Positive rheumatoid factor

Reactive arthritis is the most common cause of inflammatory arthritis in men aged 16–35. It consists of the
classic triad of arthritis, urethritis and conjunctivitis; additional extra-articular features include keratoderma
blenorrhagica, nail dystrophy, buccal erosions and circinate balanitis. Precipitants for reactive arthritis
include bacterial infections such asSalmonella or Campylobacter, and also sexually transmitted infections
such as Chlamydia, but not streptococcal throat infection. Nodules are a feature of rheumatoid arthritis.

777447225 211
Prepared by: Osama M. Alsurmi

Question 12
Which one of the following statements regarding dual energy X-ray
absorptiometry (DEXA) bone scanning is incorrect?

A. The Z-score compares the patient’s bone density to that of an age-matched control
B. A T-score of –2.5 is indicative of osteoporosis
C. DEXA is indicated in patients with an early menopause who have not received hormone
replacement therapy
D. The preferred measurement sites for bone mineral density are the hip and the lumbar spine
E. DEXA bone scanning involves the intravenous injection of radioisotope
DEXA bone scanning measures the extent to which the passage of X-ray beams through bone is attenuated.
Since this is determined by the degree of mineralisation, the relative density of the bone can be calculated.
Two scores are calculated; the T-score compares the bone mineral density of the subject to that of a healthy
young person, and the Z-score to that of an age-matched control. Both T- and Z-scores refer to numbers of
standard deviations from the mean. Osteoporosis is defined as a T-score of less than –2.5. The preferred
sites for measurement are the lumbar spine and the hip. DEXA scans should not be confused with isotope
bone scans, which involve the intravenous injection of radioisotope.

777447225 212
Prepared by: Osama M. Alsurmi

Question 13
A 24-year-old woman with sclerodactyly and symptoms suggestive of
Raynaud’s phenomenon is diagnosed with systemic sclerosis. Which one of
the following statements is true regarding her treatment?

A. Calcium channel blockers such as nifedipine may help symptoms of Raynaud’s phenomenon
B. Angiotensin-converting enzyme (ACE) inhibitors are contraindicated due to the risk of
precipitating a hypertensive renal crisis
C. Systemic steroids are useful for treating skin changes
D. H2-receptor antagonists are more effective than proton pump inhibitors for treatment of
oesophageal reflux
E. There are no effective oral treatments for pulmonary hypertension
The dihydropyridine calcium channel blockers help prevent vasospasm and are a useful treatment for
Raynaud’s phenomenon. An ACE inhibitor is an important prophylactic agent in reducing the risk of
development of a hypertensive renal crisis, particularly in patients who have diffuse cutaneous disease, as
they have the greatest risk of developing this complication. Other useful treatments include proton pump
inhibitors for oesophagitis (which is often severe) and the oral endothelin antagonist bosentan for
pulmonary hypertension. On the other hand, no agent is known to improve the cutaneous manifestations of
the disease.

777447225 213
Prepared by: Osama M. Alsurmi

Question 14
Which one of the following findings on clinical examination is characteristic
of dermatomyositis?

A. Weakness of the distal musculature


B. Tender nodules on the finger pulps
C. A violaceous rash around the eyes
D. Nail pitting
E. Peripheral oedema

Recognised cutaneous features of dermatomyositis include a violaceous rash around the eyes, Gottron’s
papules over the proximal interphalangeal and distal interphalangeal joints, periorbital oedema and
abnormal nailfold capillary loops. Weakness of proximal musculature is a cardinal feature.

777447225 214
Prepared by: Osama M. Alsurmi

Question 15
As the admitting doctor on a general medical ward, you see a patient who
has been referred from primary care and is suspected of having
dermatomyositis. Which of the following investigations (all of which may
support or refute the diagnosis) should you order first?

A. Electromyography (EMG)
B. Serum creatine kinase (CK)
C. Muscle biopsy
D. CT chest/abdomen/pelvis
E. MRI
A raised CK in the presence of the clinical features listed in the preceding question is pathognomonic of
dermatomyositis. All of the other tests, although they may be used in the investigation of dermatomyositis,
are subsidiary. EMG may confirm a myopathy and exclude a neuropathy. Muscle biopsy would typically
show fibre necrosis and an inflammatory cell infiltrate. Particularly in adult patients, and in those who are
autoantibody-negative, CT chest/abdomen/pelvis and imaging of the gastrointestinal tract should be
considered to look for an underlying malignancy (the most common associations being adenocarcinoma
and lymphoma). MRI is a useful means of identifying abnormal muscle that would be amenable to biopsy.

777447225 215
Prepared by: Osama M. Alsurmi

Question 16
A 23-year-old lady who complains of dry eyes and mouth has a positive
Schirmer’s test. Serological tests are positive for antinuclear antibody (ANA)
and rheumatoid factor. Which of the following is the most likely unifying
diagnosis?

A. Systemic lupus erythematosus (SLE)


B. Sjögren’s syndrome
C. Conjunctivitis
D. Rheumatoid arthritis
E. Dehydration

The case described fits best with a diagnosis of Sjögren’s syndrome. Although the serology would be
compatible with both SLE and rheumatoid arthritis, the description of keratoconjunctivitis sicca (dry eyes)
and xerostomia (dry mouth) and the combination of both positive rheumatoid factor and ANA are typical of
Sjögren’s.

777447225 216
Prepared by: Osama M. Alsurmi

Question 17
All of the following statements about Sjögren’s syndrome are true except
one. Which is the exception?

A. There is an association with HLA-B27


B. There is a female preponderance
C. Enlargement of the salivary glands is frequently found
D. There is an association with other autoimmune diseases such as primary biliary cirrhosis
and myasthenia gravis
E. There is an increased risk of developing lymphoma

HLA-B27 is strongly associated with ankylosing spondylitis. Sjögren’s is associated with HLA-B8 and
HLA-DR3.

777447225 217
Prepared by: Osama M. Alsurmi

Question 18
Which one of the following features is most typical of polymyalgia
rheumatica?

A. A normal erythrocyte sedimentation rate (ESR)


B. Poor response to oral corticosteroid treatment
C. Marked muscle wasting
D. Muscle stiffness and tenderness
E. Mean age of onset of 30 years

Polymyalgia rheumatica is a disease of older people; patients are almost always over 55 years, but there is a
mean age of onset of 70. The ESR is invariably elevated. There is a rapid and dramatic response to systemic
corticosteroids, with most patients experiencing relief of symptoms within 72 hours of the first dose. The
most common complaint is muscle stiffness and there is often marked muscle tenderness but no muscle
weakness. Muscle weakness and wasting suggest primary muscle disease such as polymyositis.

777447225 218
Prepared by: Osama M. Alsurmi

Question 19
A 75-year-old lady is admitted to the medical admissions unit in the early
hours of the morning with a temporal headache, visual disturbance and pain
on chewing. On examination, she has decreased visual acuity in her right
eye and tenderness over the right scalp. The erythrocyte sedimentation rate
is 95 mm/hr. Which is the most important action to take?

A. Order a CT of the brain


B. Request an ophthalmology review in the morning
C. Request a temporal artery biopsy
D. Give 30 mg of dihydrocodeine for pain relief
E. Give 60 mg of prednisolone by mouth
The history and examination findings described are absolutely classical for temporal or giant cell arteritis.
This is a large-vessel vasculitis affecting, predominantly, the branches of the temporal and ophthalmic
arteries. It is sight-threatening, and the correct treatment once the diagnosis is suspected is immediate
administration of high-dose oral corticosteroid. Such steroid treatment will not alter any histological
abnormalities obtained on temporal artery biopsy if this procedure is undertaken within 1–2 weeks.

777447225 219
Prepared by: Osama M. Alsurmi

Question 20
With regard to the anatomy of joints, which one of these statements
is incorrect?

A. The costochondral joint is an example of a fibrocartilaginous joint


B. Synovial joints always include two surfaces covered with articular cartilage
C. Articular cartilage has a rich vascular supply
D. Synovial fluid is an ultrafiltrate of plasma containing proteoglycans
E. The joint capsule is in part formed from ligaments

The major learning point here is that articular cartilage is avascular.

777447225 220
Prepared by: Osama M. Alsurmi

Question 21
Plain radiography of joints is often a helpful aid to diagnosis in bone and
joint disease. Which is the one incorrect association in the following list of
radiographic findings?

A. Bony osteophytes and subchondral sclerosis with osteoarthritis


B. Diffuse narrowing of the joint space with rheumatoid arthritis
C. Soft tissue swelling with septic arthritis
D. Looser’s zones with osteomalacia
E. Osteopenia with Paget’s disease of bone

X-rays in Paget’s disease commonly show a mixed picture of osteosclerosis and focal osteolysis, but not
osteopenia. All of the other radiological features described are accurate.

777447225 221
Prepared by: Osama M. Alsurmi

Question 22
Which one of the following statements about blood tests often used in
rheumatology is incorrect?

A. A high titre of rheumatoid factor at first presentation of rheumatoid arthritis is a poor


prognostic factor
B. Anti-double-stranded DNA (anti-dsDNA) antibodies are highly specific for systemic lupus
erythematosus (SLE)
C. Antinuclear antibody (ANA) is a highly sensitive test for SLE
D. The serum creatine kinase is often elevated in dermatomyositis
E. Serum biochemistry is usually normal in osteomalacia
Osteomalacia is commonly associated with low serum calcium and phosphate, accompanied by elevated
alkaline phosphatase. In contrast, osteoporosis is associated with normal serum biochemistry. Options B
and C highlight the difference in the sensitivity and specificity of tests. ANA has a high sensitivity for lupus
(almost 100%), so if a person tests negative for ANA this virtually excludes the diagnosis. (High sensitivity
is a desirable quality in a good screening test.) Anti-dsDNA antibodies, however, provide a highly specific
test for lupus, meaning that if a person tests positive for anti-dsDNA antibodies, then the diagnosis is
almost certainly lupus. Unfortunately, anti-dsDNA has low sensitivity for lupus.

777447225 222
Prepared by: Osama M. Alsurmi

Question 23
In the investigation of a patient who has suspected Henoch–Schönlein
purpura, which of the following investigations is least likely to contribute to
the diagnosis?

A. Dipstick and microscopy of urine for red cells


B. Urea and electrolytes (U&E)
C. Full blood count (FBC)
D. Chest X-ray
E. Skin biopsy

A chest X-ray is the least likely to aid diagnosis. A skin biopsy, although invasive, would confirm the
diagnosis by demonstrating on immunofluorescence IgA deposition in the vessel walls. The serum IgA
levels are also often raised. Henoch–Schönlein purpura may cause an acute nephritis, leading to haematuria
and/or renal failure, thus U&E and urinalysis are important. An FBC should be checked in all cases of
purpura to exclude the possibility of thrombocytopenia (low platelet count).

777447225 223
Prepared by: Osama M. Alsurmi

Question 24
A number of weeks after commencing a new tablet for her blood pressure, a
49-year-old lady with hypertension presents to her general practitioner with
a photosensitive rash affecting her face and hands, and new-onset
arthralgia. The GP also finds microscopic haematuria on urinalysis and
suspects drug-induced lupus. Which of the following medications is most
likely to have caused this presentation?

A. Atenolol
B. Lisinopril
C. Minoxidil
D. Hydralazine
E. Nifedipine

Hydralazine, along with procainaminde, isoniazid, minocycline and chlorpromazine, can cause a drug-
induced lupus syndrome that commonly presents as rash and arthralgia. Renal involvement is less common
in drug-induced lupus than in primary lupus but can occur.

777447225 224
Prepared by: Osama M. Alsurmi

Question 25
Which one of the following conditions is least likely to occur as a complication
of Marfan’s syndrome?

A. Lens dislocation
B. Spontaneous pneumothorax
C. Aortic dissection
D. Mitral valve prolapse
E. Pulmonary embolism

Cardiovascular complications are the leading cause of death in patients with Marfan’s, particularly aortic
dissection which accounts for 27–48% of deaths. There is no increased risk of pulmonary embolism.

777447225 225
Prepared by: Osama M. Alsurmi

Question 26
Which one of the following agents is not classified as a disease-modifying
antirheumatic drug (DMARD)?

A. Sulfasalazine
B. Naproxen
C. Hydroxychloroquine
D. Sodium aurothiomalate (gold)
E. Methotrexate

Disease-modifying drugs improve symptoms, lower levels of inflammatory markers such as erythrocyte
sedimentation rate and C-reactive protein, and are thought to modify the underlying disease process in
rheumatoid arthritis. Although naproxen may improve symptoms and reduce joint swelling, it is not thought
to modify the disease process and is not classified as a DMARD.

777447225 226
Prepared by: Osama M. Alsurmi

Question 27
All of the following statements except one are true of polyarteritis nodosa.
Which is the exception?

A. Skin manifestations include palpable purpura and livedo reticularis


B. There is an association with hepatitis B infection
C. It is a large-vessel vasculitis
D. Peripheral nerve palsies may be associated
E. Severe hypertension is a recognised complication

Polyarteritisnodosa (PAN) is a rare vasculitis of small and medium-sized (not large-conduit) vessels; it
predominately affects men, is negative for antineutrophil cytoplasmic antibodies (ANCA) and may have a
number of manifestations, including arteritis of the vasa nervorum, leading to peripheral neuropathy,
abdominal pain, fever and severe hypertension secondary to renal infarctions. Hepatitis B is a risk factor for
development of PAN, and incidence of PAN is higher in places such as Alaska, where the infection is
endemic.

777447225 227
Prepared by: Osama M. Alsurmi

Question 28
A 12-year-old boy is brought to clinic by his mother with a rash on his legs
that has appeared over the last 3 days. He also complains of pain in his
knees and hands, and in the last 24 hours has developed central abdominal
pain. Prior to this episode he was entirely well, apart from what his mother
describes as a ‘cold’ 2 weeks ago. On examination, you find a palpable
purpuric rash over the buttocks and lower limbs. What is the likeliest
diagnosis?

A. Still’s disease
B. Henoch–Schönlein purpura
C. Kawasaki disease
D. Rheumatic fever
E. Juvenile idiopathic arthritis

The combination of arthritis, abdominal pain and purpura in a patient of this age suggests a diagnosis of
Henoch–Schönlein purpura. The condition often follows an upper respiratory tract infection.

777447225 228
Prepared by: Osama M. Alsurmi

Question 29
A 76-year-old man with type 2 diabetes and longstanding osteoarthritis of
the knees and hips presents to the hospital accident and emergency
department with a 72-hour history of progressive pain and swelling of his
right knee. He was recently treated by his general practitioner for an
episode of cellulitis. On examination, his temperature is 38.5oC and there is
a warm, tense effusion with marked tenderness and overlying erythema.
Which of the following is the most likely diagnosis?

A. Haemarthrosis
B. Trauma
C. Septic arthritis
D. Acute gout
E. First presentation of rheumatoid arthritis
Although all of the above may cause an acute monoarthritis, septic arthritis is the most likely cause, given
the progressive pain, overlying erythema, pyrexia, recent episode of cellulitis (representing a possible portal
of entry for infection) and presence of risk factors (diabetes mellitus, pre-existing joint disease, age). With
crystal synovitis (urate, pyrophosphate), pain is at its maximum within the first day; progressive pain and
overlying erythema should always suggest sepsis.

777447225 229
Prepared by: Osama M. Alsurmi

Question 30
A 70-year-old lady who has recently noticed some hearing difficulty is found
to have a greatly increased plasma alkaline phosphatase concentration with
normal calcium, phosphate and parathyroid hormone (PTH) levels. What is
the most likely diagnosis?

A. Osteomalacia
B. Rickets
C. Renal osteodystrophy
D. Paget’s disease
E. Osteoporosis

Paget’s disease is a common condition characterised by increased and disorganised bone remodelling. The
biochemical picture is that of a normal calcium, phosphate and PTH, in conjunction with a raised alkaline
phosphatase. Osteoporosis (in the absence of a fracture) does not cause any biochemical abnormality. The
fact that the lady has been complaining of difficulty in hearing may be significant, as deafness is a clinical
feature of Paget’s disease.

777447225 230
Prepared by: Osama M. Alsurmi

Question 31
A gentleman of 70 with longstanding rheumatoid arthritis is found on a
routine blood test to have a white cell count of less than 1 × 109/L. He
complains of abdominal pain, and on examination you find generalised
lymphadenopathy and splenomegaly. Which of the following is the most
likely diagnosis?

A. Malaria
B. Acute lymphatic leukaemia (ALL)
C. Felty’s syndrome
D. Portal hypertension secondary to alcoholic liver disease
E. Overwhelming septicaemia

Although an enlarged spleen may potentially be found in any of the above conditions, Felty’s syndrome
refers to the association of splenomegaly and neutropenia with rheumatoid arthritis and is therefore the
most likely diagnosis here. It has a peak incidence in the 50–70-year-old age group, typically occurring in
patients with longstanding disease and a positive rheumatoid factor. In addition to splenomegaly, clinical
features include recurrent infections, weight loss, leg ulcers and lymphadenopathy.

777447225 231

Anda mungkin juga menyukai